Chapter 46, bowel elimination, Fundamentals of Nursing Exam IV Study questions (Urinary Elimination, Fecal Elimination, Oxygenation, Fluid and Electrolytes), Urinary elimination

अब Quizwiz के साथ अपने होमवर्क और परीक्षाओं को एस करें!

Factors to decrease UTI

-avoid the routine use of catheters;only when indicated -collaborate with heatlh care providers to remove catheters when medical indications no longer exist -suggest noninvasive continent devices such as condom catheters to reduce the risk -need to be patient advocate by taking an active role in monitoring duration of treatment

Which medication has this action on the GI system (1091): Suppresses peristalsis and decreases gastric emptying.

Dicyclomine HCl (Bentyl)

A client is scheduled for a plain film x-ray of the kidney, ureter, and bladder. What preliminary preparations should this client take?

For getting a plain film of kidney, ureter, and bladder there is no specific preliminary preparation that needs to be done.

Which enema type is contraindicated for dehydrated pt. and young infants?

Hypertonic. It is dehydrating (Fleet enema is an ex.)

A nurse is caring for a client who has had diarrhea for the past 4 days. When assessing the client, the nurse should expect which of the following findings? (Select all that apply.) ___Bradycardia ___Hypotension ___Fever ___Poor skin turgor ___Peripheral edema ___Abdominal cramping

Hypotension Fever Poor skin turgor Abdominal cramping Prolonged diarrhea leads to dehydration, which is characterized by tachycardia, hypotension, fever, lethargy, poor skin turgor, and abdominal cramping. Peripheral edema is more likely to be caused by a fluid overload rather than a fluid deficit. Chapter 45; Page 1096, Box 46-3

What is the difference between distention caused by gas and distention caused by fluid?

If fluid is the source of the problem, the flanks will bulge too. But if it's gas, they don't bulge, just the abdomen protrudes

Which medication has this action on the GI system (1091): Causes discoloration of the stool (black), nausea, vomiting, constipation (diarrhea is less commonly reported), and abd cramps.

Iron

does the insertion of an NG tube require sterile technique

no

can all ostomies be irrigated?

nope. Just colostomies

Should you use sterile technique when giving enemas?

nope. The colon isn't sterile, so it's not necessary

what is the safest to use for an enema cleansing

normal saline

abnormal characteristics of stool: odor

noxious change-blood in feces or infectin

what is expulsion of feces assisted by

contraction of abdominal muscles and diaphragm contraction of the levator ani muscles of pelvic floor thigh flexion increases pressure sitting position increases pressure

enteroclysis

contrast material is introduced to jejunum, allowing entire small intestine to be studied -prep 24 hours of clear liquid diet and colon cleansing such as GoLytely or enemas until clear

food that helps urinary infection

cranberry, apple, and prune juices

The nurse assesses that the patient has a full bladder, and the patient states that he or she is having difficulty voiding. The nurse would teach the patient to: Use the double-voiding technique. Perform Kegel exercises. Use Credé's method. Correct Keep a voiding diary.

credes method pressure is put on the suprapubic area with each attempted void. the maneuver promotes bladder emptying by relaxing the urethral sphincter

melena

abnormal black sticky stool containing digested blood-indicative of gastro intestinal bleeding

What does this indicate? White or clay colored stool

absence of bile

computerized tomography scan

an x-ray film examination of the body from many angles uses a scanner analyzed by a computer -preparation is usually NPO -the patient needs to lie very still; if claustrophobia use light sedation

barium enema with air contrast

an x-ray film examn uses an opawue contrast medium and air that outlines the colon and rectum to examine the lower GI tract prep: NPO after midnight, bowel prep such as magnesium citrate

Upper gastrointestinal barium swallow

an xray exam using a opaque contrast medium- structure and motility of the upper GI tract and pharynx, esophagus, and stomach prep: NPO After midnight the night before the exam remove all jewelry or other metallic objects after the test patient needs to increase fluids to facilitate passage barium

list medications that can cause constipation

anticholinergics, antispasmodics, anticonvulsants, antidepressents, antihistamines, antihypertensives, antiparkinson drugs, diuretics, antacids, iron supplements, calcium supplements, opoids

list the parts of the colon

ascending transverse descending sigmoid

Before collecting a stool sample for occult blood, the nurse instructs the nursing assistive personnel to:

ask to void Emptying the urinary bladder before collecting the stool sample prevents contamination of the specimen.

risk factors for developing colon cancer

obesity inactivity >50 family history of colon cancer ethnic background: Jew of eastern European descent race: African Americans diet: high intake of animal fats, and low in fruits and veggies smoking and alcohol intake diabetes

if you have observable perstaltitic waves what does it usually indicate

obstruction

A client reports passing narrow pencil-shaped stools over the past few days. Which conditions should the client be evaluated for? Select all that apply.

obstructions and rapid peristalsis

plastic catheterizaiton

only for intermittent use

A nurse is caring for a client with malabsorption syndrome. What changes in bowel elimination is the client likely to report? Select all that apply. Pale stools Black, tarry stools Clay-colored stools Increased flatulence Fat and mucus in stools

pale and fat/mucus in stools A client with malabsorption syndrome is unable to digest fats. Therefore, oily fat and mucus present in the stools give the stools a pale color. Black or tarry stools can be caused by consumption of iron preparations. One of the most common causes of increased flatulence is swallowing air from activities such as chewing gum, drinking carbonated beverages, eating rapidly, or sucking through straws. Biliary obstruction or a lack of bile production may cause the stools to become white, gray, or clay-colored.

who would you not want to have valsalva maneuver happen with

patients with cardiac dysrrthmias. can cause heart attack

Physical activity and bowel elimination

physical activity promotes peristalis and immobilization depress it.

suprapubic cath

placement of cath rthrough the abdominal wall into the bladder. must remain in pt at all times. monitor I&O carefully, urine charcterisitics, nd observe for signs of infection.

upper endoscopy

prep: NPO After midnight the night before the exam remove all jewelry or other metallic objects after the test patient needs to increase fluids to facilitate passage barium light sedation

what is the most effective treatment for diarrhea

prescriptive opiates- 1. codeine phosphate 2. opium tincture (paregoric) 3.diphenoxylate (lomotil) they slow intestinal muscle tone to slow passage of feces

The nurse who works in a medical surgical unit observes that women have a higher incidence of urinary tract infections than men. Why is this?

shortened urethra absence of prostatic secretions proximity of urethral meatus to anus The increased susceptibility to urinary tract infection among females is due to their short urethra, making the upward transmission of microorganisms easier. Women also lack prostatic secretions, which have antibacterial properties. The proximity of the urethral meatus to the anus among females leads to the spread of microorganisms from the anus to the urethra. The presence or absence of testosterone does not have any impact on urinary function. It is responsible for sex characteristics. Kidney structure does not vary between men and women.

how to pick catheter size?

sizse of pt urthral canal. french system: larger catheter larger the guage number

Which portion of the GI tract is affected here? These are normal age-related changes: (p 1090) Decrease nutrient absorption caused by fewer absorbing cells.

sm. intestine

what are the 2 types of NG tubes

small bore- medication administration and enteral feedings large bore- (12 Fr & above)-gastric decompression or removal of gastric secretions (Levin &Salem sump tubes are most common) it connects to a drainage bag or intermittent suction device

where does segmentation and peristaltic mvmt take place

small intestine

A client needs a bowel preparation before a procedure. Which medication is appropriate in managing this client? Psyllium Polycarbophil Methyl cellulose Sodium phosphate

sodium phosphate Sodium phosphate is meant for acute emptying of the bowel and is suitable for managing the client with acute constipation. Psyllium, polycarbophil, and methyl cellulose as they are bulk- forming agents are the drugs of choice for managing chronic constipation, rather than acute constipation.

TYPE 5 stool

soft blobs with clear-cut edges (passed easily)

what is the primary position for elimination

squatting

The nurse is reviewing laboratory results for a client and notices the urine tested positive for ketones. What underlying factors may lead to the presence of urinary ketone bodies? Select all that apply. 1. epilepsy 2. starvation 3.dehydration 4. hyperthyroidism 5. uncontrolled diabetes mellitus

starvation, dehydration, and diabetes Ketones are produced as a by-product when the body uses fat for energy production. When a client is not taking in adequate amounts of carbohydrate, such as in starvation, the body uses other sources for energy. Dehydration can also lead to ketonuria. A client with uncontrolled diabetes mellitus breaks down fatty acids for energy. Epilepsy and hyperthyroidism are not associated with the presence of ketone bodies in urine. Epilepsy is a disease that affects the nervous system, and hyperthyroidism affects the endocrine system.

intermittent cateterization

straight single-use cathether long enough to drain the bladder- 5 to 10 minutes after finished immediately withdraw cat increases risk for infection and trauma pt with spinal cord injuries or MS

restorative care

strengthening pelvic floor muscles bladder retraining habit training self-cath maintence of skin integrity promotion of comfort

bisacodyl and castor oil

suiable for preparing the bowel for carrying out diagnostic procedures

docusate calcium

suitable for short term therapy for relieving straining during defecation

Dicyclomine HcL (Bentyl)

suppresses peristalsis & < gastric emptying

alternative to urethral catheterization

suprapubic catheterization condom catheter

Constipation is not a disease, but a __.

symptom

paralytic ileus

temporary stopping of peristalsis

when does the rectum get feces into it

the rectum is the last area. it converts fecal matter into its final form. Normally the rectum is empty of feces until just before defecation

How many times does an adult normally defecate?

daily or 2-3 times a week. It is abnormal if it is occurring more than 3 times a day or less than once per week

what patients are most at risk for impaction

debilitated, confused, or unconscious patients

what does a nasogastric tube (NG) do?

decompresses the GI tract. it is inserted thru patients nasopharynx into stomach

do anesthetics increase or decrease peristalsis

decrease

anticholinergics(atropine, robinul)

decrease gastric motility=can cause constipation

does depression increase or decrease peristalsis

decreases

Histamine antagonist

decreases HcL and interfere with digestion

Fast peristalsis = ?

diarrhea

what is diarrhea a sign of disorder involving

digestion, absorption, and secretion

what would be the last resort to remove a fecal impaction

digital removal

Iron

discoloration of stool, nausea, vomit, constipation, & cramps

what often causes a strong urge to defecate in older adults

dulcolax

what part of the small intestine absorbs most of the nutrients and electrolytes

duodenum & jejunum

what are the 3 sections of the small intestine

duodenum- 8-11 inches-processes the chime from stomach jejunum-8 ft long-absorbs carbs and proteins ileum-12 ft long- absorbs H20, fats, vitamins, iron, & bile salts

when is the strongest event of mass peristalsis

during the hour after mealtime

when is amylase elevated

elevated in pancreas abnormalities of the pancreas -inflammation or tumors -cholecystitis -necrotic bowel -diabetic ketoacidosis

what are lab tests fsor bowel function

total bilirubin(0.3-1)mg alkaline phosphate (30-120) units carcinoembryonic antigen(less than 5), amylase (60-120) elevated amylase indicates abnormalities in pancreas more than 5 carcinoembroyonic antigen indicates cancer or inflammation of GI tract

Spastic constipation presents with stools with

translucent mucus

bladder retraining

treating pt with stress incontinence and decreased urge to void goal is reduce voiding frequency and perhaps the bladder capacity

kegal excercises

treating stress incontinence, OAB, mixed cause of urinary incontinence -pt need to be alert and motivated -continued use -learn the technique while voiding-->then practice not voiding

hemorrhoids

underlying bulging vein that is usually purplish discoloration and can be hardened

latex catheters

up to 3 weeks, be aware of allergies

maintenance of skin integrity

urine on skin causes encrustations that fosters breakdown 1. washing with mild soap and water to remove 2. body lotion keeps skin moisturized 3. petroleum-based ointments provide barrier to the urine

Sodium polystryrene sulfonate (Kayexalate)?

used to treat pt. with dangerous levels of potassium. exchanges sodium ions for potassium ions

who are particularly susecptible to fluid and electrolyte imbalances

very young and elderly

what does intrinsic factor absorb

vitamin B12

TYPE 7 stool

watery, no solid pieces. Entirely liquid

does early ambulation help with defecation

yes

are suppositories effective

yes but oral is most common method used

Can a guaiac fecal occult blood test be delegated to a NAP

yes, but nurse must evaluate if bright red blood is found. patient needs to void before test also

Do you need a health care provider's order to remove fecal impaction? can you delegate it?

yes. No. CNAs cannot remove fecal impactions

Which medication has this action on the GI system (1091): Causes GI irritation that increases the incidence of bleeding with serious consequences to older adults; rectal bleeding is often observed with GI irritation.

Nonsteroidal anti-inflammatory drugs

Which medication has this action on the GI system (1091): Slow peristalsis and segmental contractions often resulting in constipation.

Opioid analgesics

After an enema is infused, what position should the patient be in and why?

Patient should turn from the left lateral to dorsal recumbent, over to the right lateral position. The position change ensures that fluid reaches the large intestine. Chapter 46; Page 1107

Which laxative/cathartic type is described: Irritates intestinal mucosa to increase motility. Decreases absorption in sm. bowel and colon. Used to prepare bowel for diagnostic procedures. Causes severe cramping. Causes fluid and electrolyte imbalances. Not for long-term use

Stimulant Cathartics. Bisacodyl (Dulcolax) Castor oil Casanthranol (Peri-colace) Senna (Ex-lax, senokot)

what are the 3 functions of the large intestine

1. absorb 2. secrete 3. eliminate

when does a child develop neuromuscular control of defecation

2-3 yrs

You would hear an increase in pitch or tinkling sounds with ___.

abdominal distention

Bristol Stool Scale TYPE 1

seperate hard lumps, like nuts (hard to pass)

What are the pre-test and post-test procedures should a nurse follow while managing a client who is scheduled for a cystoscopy? Select all that apply.

-monitor intake andoutput -inform the client that he may have voiding difficulty post procedure -inform the client that he may pass red-tinged urine post procedure Fluid intake and urine output should be monitored post cystoscopy to determine obstruction or trauma to the urinary system. After the procedure, the client may have difficulty voiding or have red or pink urine because of trauma to the urethral or bladder mucosa. Fluids should be encouraged to promote urine formation and prevent infection. Cystoscopy can be used as a therapeutic procedure and may have untoward effects.

Normal value for bilirubin?

0.3-1 mg/dL

how much water and sodium is absorbed from the colon every 4 hours

1 gallon of H20 and an ounce of Na

A nurse determines that a fracture bedpan should be used for the patient who: 1.has a spinal cord injury. 2.Is on bedrest. 3.Has dementia. 4.Is obese

1* A fracture bedpan has a low back that promotes function of the patient's lower back while on the bedpan. 2. A regular bed pan is appropriate for this patient 3.A regular bed pan is appropriate for this patient 4.A regular bed pan is appropriate for this patient

The nurse is teaching kidney function to a group of nursing students. Which of the following statements apply to kidney function? 1.The kidneys produce several substances vital for maintenance of blood pressure. 2 The kidneys produce several substances vital to bone mineralization. 3 A nephron is a functional unit of the kidney and helps in urine formation. 4 The kidneys filter waste products of metabolism and excrete them in the urine. 5 The kidneys produce several substances vital to white blood cell (WBC) production

1, 2, 3, 4 The kidneys produce several substances vital for maintenance of blood pressure. Correct 2 The kidneys produce several substances vital to bone mineralization. Correct 3 A nephron is a functional unit of the kidney and helps in urine formation. Correct 4 The kidneys filter waste products of metabolism and excrete them in the urine. 5 The kidneys produce several substances vital to white blood cell (WBC) production

A student nurse is learning to auscultate the abdomen for bowel sounds. What should the student nurse know about the bowel sounds before starting to auscultate? Select all that apply. 1.Normal bowel sounds occur every 5-15 2.Each bowel sound lasts one to several seconds. 3.An increase in pitch or a tinkling sound indicates bowel distention. 4.Hypoactive sounds may occur in small intestine obstructions. 5. Hyperactive sounds may occur in a paralytic ileus.

1,2,3 Bowel sounds can be assessed by auscultating the four quadrants of the abdomen. It is considered normal if the bowel sounds occur every 5-15 seconds. Each bowel sound may last one to several seconds. In bowel distention, an increase in pitch or a tinkling sound can be heard. Hypoactive sounds are less than 5 sounds per minute and may occur in paralytic ileus. Hyperactive bowel sounds or 35 or more sounds per minute occur when there is an intestinal obstruction.

A client's midstream urine sample is being collected for laboratory investigation. Arrange the steps that a nurse should follow after the sample has been collected in the container.

1. Replace the cap on the container. 2. Clean urine from the exterior surface of the container. 3. Attach a label on the side of the container. 4. Attach a laboratory requisition to the specimen bag. 5. Remove gloves and perform hand hygiene.

A practitioner orders a return flow enema (Harris flush drip) for an adult patient with flatulence. When preparing to administer this enema The nurse compares the steps of a return flow enema with cleansing enemas. What should the nurse do that is unique to a return flow enema? 1. Lubricate the last 2 inches of the rectal tube. 2Insert the rectal tube about 4 inches into the anus. 3. Raise the solution container about 12 inches above the anus. 4.Lower the solution container after instilling about 150 mL of solution

1. All rectal tube should be lubricated to facilitate entry of the tube into the anus and rectum and prevent mucosal trauma. 2.The anal canal is 1 to 2 inches long. Inserting the rectal tube 3 to 4 inches ensures that the tip of the tube is beyond the anal Sphincter. This action is appropriate for all types of enemas. 3. The solution container should be raised no higher than 12 inches for all enemas; this allows the solution to instill slowly, which limits discomfort and intestinal spasms 4.* lowering the container of solution create a siphon effect that pulls the instilled fluid back out through the rectal tube into the solution container. The return flow promotes the evacuation of gas from the intestines. This technique is used only with a return flow enema.

A nurse is caring for a debilitated female patient with nocturia. Which nursing intervention is the priority when planning to meet this patient's needs? 1.Encouraging the use of bladder training exercises 2. Providing assistance with toileting every four hours 3. Positioning a bedside commode near the bed 4. Teaching the avoidance of fluid after 5 PM

1. Although this should be done, it is not the priority. 2. This may be too often or not often enough for the patient. Care should be individualized for the patient 3.* The use of a commode requires less energy than using a bedpan and is safer than walking to the bathroom. Sitting on the commode uses gravity to empty the bladder fully and thus prevent urinary stasis 4. Fluids may be decreased during the last two hours before bedtime, but they should not be avoided completely after 5 PM. Some fluid intake is necessary for adequate renal perfusion.

A nurse must measure the intake and output (I & O) for a patient who has a urinary retention catheter. Which equipment is most appropriate to use to accurately measure urine output from a urinary retention catheter? 1.Urinal 2.graduate, 3.large syringe, 4.urine collection bag

1. Although urinals have volume markings on the side, usually they occur in 100 mL increments that do not promote accurate measurements 2.* A graduate is a collection container with volume markings usually at 25 mL increments that promote accurate measurements of urine volume 3. This is impractical. A large syringe is used to obtain a sterile specimen from a retention catheter (Foley catheter) 4. A urine collection bag is flexible and balloons outward as urine collects .In addition, the volume markings are at 100 mL increments that do not promote accurate measurements.

A nurse is applying to a warm compress. What should the nurse explain to the patient is the primary reason why heat is used instead of cold? 1.Minimizes muscle spasms 2.Prevents hemorrhage. 3.Increases circulation. 4.Reduces discomfort

1. Both heat and cold relax muscles and thus minimize muscle spasms. There is no advantage to using heat over cold. 2. Heat does not prevent hemorrhage; heat causes vasodilation, which promotes hemorrhage. 3.* heat increases the skin surface temperature, promoting vasodilation, which increases blood flow to the area. Cold has the opposite effect: it promotes vasoconstriction, which decreases blood flow to the area. 4. Both heat and cold can reduce discomfort. Cold reduces discomfort by numbing the area, slowing the transmission of pain impulses, and increasing the pain threshold. Heat reduces the discomfort by relaxing the muscles

A nurse has identified that the patient has overflow incontinence. What is a major factor that contributes to this clinical manifestation? 1.Coughing 2.mobility deficits 3.prostate enlargement 4.urinary tract infection

1. Coughing, which raises the intro abdominal pressure, is related to stress incontinence, not overflow incontinence. 2. Mobility deficits, such as spinal cord injuries, are related to reflex incontinence, not overflow incontinence. 3.* an enlarged prostate compresses the urethra and interferes with the outflow of urine, resulting in urinary retention. With urinary retention, the pressure with in the bladder builds until the external urethral sphincter temporarily opens to allow a small volume (25-60mL) of urine to escape(overflow incontinence) 4. Urinary tract infections are related to urge incontinence, not overflow incontinence.

A nurse teaches a patient about pursed lip breathing. The nurse identifies that the teaching is affected when the patient says it's purpose is to: 1.precipitate coughing. 2.Help maintain open airways. 3.Decrease intrathoracic pressure. 4.Facilitate expectoration of mucus

1. Deep breathing and huff coughing, not pursed lip breathing, stimulate effective coughing. 2.* pursed lip breathing involves deep inspiration and prolonged expiration against slightly closed lips. The pursed lips create a resistance to the air flowing out of the lungs, which prolongs exhalation and maintains positive airway pressure, thereby maintaining an open airway and preventing airway collapse 3. Pursed lip breathing increases, not decreases intrathoracic pressure. 4.The huff coughing stimulates the natural cough reflex and is effective for clearing the central air ways of sputum. Saying the word huff with short forceful exhalations keeps the glottis open, mobilizes sputum, and stimulates a cough.

List the order in which an abdominal assessment should occur.

1. Inspection 2. Auscultation 3. Palpation 4. Percussion

A patient with the diagnosis of diverticulosis is advised to eat a diet high in fiber. What should the nurse recommend that the patient eat to best increase the bulk and fecal material? 1.Whole wheat bread. 2.White rice. 3.Pasta. 4.Kale.

1. One slice of whole wheat bread contains only 1.5 g of dietary fiber. 2.A serving of a 1/2 cup of white rice contains only 0.8 g of dietary fiber. 3.A serving of 3 1/2 ounces of cooked pasta contains only 1.6 g of dietary fiber 4.* Kayle is an excellent source of dietary fiber. A serving of 3 1/2 ounces of kale contain 6.6 g of dietary fiber

What are some environmental or physical conditions that may cause a pt. to suppress defecation (which could lead to constipation)?

1. Pain during defecation 2. Embarrassing situations (such as sharing a bathroom with another pt with different habits)

The nurse teaches a patient how to use an incentive spirometer. What patient outcome will support the conclusion that the use of the incentives spirometer was effective? 1.Supplemental oxygen use will be reduced. 2.Inspiratory volume will be increased. 3.Sputum will be expectorated. 4.Coughing will be stimulated.

1. Patients who use an incentive spirometer may or may not be receiving oxygen 2.* an incentive spirometry or provides a visual goal for and measurement of inspiration. It encourages the patient to execute and maintain a sustained inspiration. A sustained inspiration opens airways, increases the inspiratory volume, and reduces the risk of atelectasis 3. Although sputum maybe expectorated after the use of an incentive Iran mature, this is not the primary reason for its use. 4.Although the deep breathing associated with the use of an incentive barometer may stimulate coughing, this is not the primary reason for its use.

mrs. grayson had stress incontinence for the past 2 years. embarrassed about her problems n not spoken about it, would like to regain urinary control, weights 200 pounds and 5''1

1. Teach Mrs. Grayson supportive measures to reduce intraabdominal pressure such as losing weight and avoiding heavy lifting. Other measures that will help her include initiating a bladder and habit training program, maintaining a toileting schedule, having her use double voiding, and encouraging her to reduce caffeinated beverages from her diet. 2. Mrs. Grayson is experiencing urinary tract infection (UTI). Residual or retained urine in her bladder causes the urine to become more alkaline and is an ideal site for microorganism growth. Encourage Mrs. Grayson to increase her fluid intake to 2200 to 2700 mL/day. Teach her to empty her bladder completely. Ensure that she continues to use good perineal hygiene. 3. Strategies to teach Mrs. Grayson to prevent infection include maintaining good hand hygiene, cleaning from front to back of perineum, performing catheter care, taking care not to raise bag higher than bladder (preventing reflux of urine). Instruct Mrs. Grayson to maintain an adequate fluid intake and a closed urinary drainage system. Also tell Mrs. Grayson to prevent kinks in catheter tubing.

A client who has malabsorption syndrome asks the nurse about the process of nutrient absorption. What response should the nurse give the client? Select all that apply. 1 The ileum absorbs fat and bile salts. 2 The duodenum absorbs iron and vitamins. 3 The jejunum absorbs carbohydrates and proteins. 4 Nutrients are absorbed into the blood vessels. 5 Plant fiber undergoes absorption in the small intestine.

1. The ileum absorbs fat and bile salts 3. The jejunum absorbs carbs and proteins 4. nutrients are absorbed into the blood vessels Fats and bile salts undergo absorption in the ileum, whereas the carbohydrates and proteins undergo absorption in the jejunum. Nutrients that cross the mucosal barrier of the intestine are absorbed into the lymph fluids or blood vessels. Iron and vitamins are absorbed in the ileum and not the duodenum. Plant fiber is not digested in the small intestine; the undigested fiber is emptied into the cecum.

A nurse discourages a patient from straining excessively when attempting to have a bowel movement. What physiological response primarily may be prevented by avoiding straining on defecation? 1.Incontinence, 2.dysrhythmias, 3.fecal impaction, 4.rectal hemorrhoids

1. The loss of the voluntary ability to control the passage of fecal or gaseous discharges through the anus is caused by impaired functioning of the anal sphincter or it's nerve supply, not straining on defecation 2.* straining on defecation requires the person to hold the breath while bearing down. This maneuver increases the intrathoracic and intracranial pressures, which can precipitate dysrhythmias, brain attack, and respiratory difficulties; all of these can be life-threatening. 3. Fecal impaction is caused by prolonged retention and the accumulation of fecal material in the large intestine, not straining on defecation 4. Although straining on defecation can contribute to the formation of hemorrhoids, this is not the primary reason straining on defecation is discouraged. Hemorrhoids, although painful, are not life-threatening.

A practitioner orders chest physiotherapy with percussion and vibration for a newly admitted patient. Which information obtained by the nurse during the health history should alert the nurse to question the practitioners order? 1.Emphysema. 2.Osteoporosis. 3.Cystic fibrosis. 4.Chronic bronchitis

1. These are appropriate interventions for a patient with emphysema. Emphysema is a chronic pulmonary disease characterized by an abnormal increase in the size of air spaces distal to the terminal bronchioles with destructive changes in their walls 2.* implementing the practitioners order may compromise patient safety because percussion and vibration in the presence of osteoporosis may cause fractures. Osteoporosis is an abnormal loss of bone mass and strength. 3. These are appropriate interventions for a patient with cystic fibrosis cystic fibrosis causes widespread dysfunction of the exocrine glands. It is characterized by thick, tenacious secretions in the respiratory system that block the bronchioles, creating breathing difficulties 4. These are appropriate interventions for a patient with chronic bronchitis. Bronchitis is an inflammation of the mucous membranes of the bronchial airways.

A patient's urine is cloudy, is amber, and has an unpleasant odor. What problem may this information indicate that requires the nurse to make a focused assessment? 1.Urinary retention, 2.urinary tract infection, 3.Keytone bodies in the urine, 4.high urinary calcium level,

1. These clinical manifestations do not reflect urinary retention. Urinary retention is evidenced by supra pubic distention and lack of voiding or small, frequent voiding ( overflow incontinence) 2* The urine appears concentrated (amber)and cloudy because of the presence of bacteria, white blood cells, and red blood cells. The unpleasant odor is caused by pus in the urine (pyuria) 3. These clinical manifestations do not reflect Keytone bodies in the urine. A reagent strip dipped in urine will measure the presence of Keytone bodies 4. These clinical manifestations do not reflect excessive calcium in the urine. Urine calcium levels are measured by assessing a 24 hour urine specimen.

What should the nurse do first if a patient is choking on food? 1.Apply sharp for thrusts over the patients xiphoid process. 2.Determine if the patient can make any verbal sounds. 3.Hit the middle of the patients back firmly. 4.Sweep the patient's mouth with a finger

1. Thrusts to the xiphoid process may cause a fracture that may result in a pneumothorax 2.* when a person is choking on food, the first intervention is to determine if the person can speak because the next intervention will depend on if it is a partial or total airway obstruction. With a partial airway obstruction, the person will be able to make sounds because some air can pass from the lungs through the vocal chords. In this situation the person's own efforts open parentheses gagging and coughing) should be allowed to clear the airway. With a total airway obstruction, the person will not be able to make a sound because the airway is blocked and the nurse should immediately initiate the abdominal thrust maneuver (Heimlich maneuver)

An older adult presents with urinary frequency due to cystitis. What nursing instructions are helpful to this client? Select all that apply.

1. advise cranberry juice 2. encourage pt to increase fluid intake 3. discourage intake of caffeine Cranberry juice decreases bacterial infections of the bladder and urinary frequency due to cystitis. Clients should be encouraged to increase fluid intake to at least six to eight glasses a day unless medically contraindicated. Bacterial growth is minimal in diluted urine. Incomplete urination tends to increase the frequency, and also increase the risk of infection. Coffee, tea, cola, and alcohol intake should be discouraged since these have a diuretic effect and may increase urinary frequency. Routine use of indwelling catheters should be avoided as it may cause urinary infection. Restricting fluid intake does not decrease urinary incontinence severity or frequency.

Mr. Ryan Kelter is a 33-year-old Caucasian who lives in an acute rehabilitation center. He was injured in a motorcycle accident that caused a spinal cord injury (SCI). As a result of the SCI, he has neurogenic bladder that prevents him from fully emptying his bladder. Because of this, he needs to be straight catheterized several times a day. Beth is the student nurse assigned to Mr. Kelter. She understands the importance of keeping him on his bladder schedule to prevent a urinary tract infection (UTI). 1. Beth enters Mr. Kelter's room after lunch to perform straight catheterization. List in order the steps Beth takes to perform straight catheterization on Mr. Kelter. A. Lubricate the catheter. B. Clean penis with dominant hand. C. Apply sterile gloves. D. Advance catheter into penis. E. Apply fenestrated drape. F. Hold penis with nondominant hand. G. Ask patient to bear down. H. Coil catheter in dominant hand. 2. Beth should advance the catheter ________ to _______ inches or until urine flows out of it. 3. As Beth inserts the catheter into Mr. Kelter's penis, she feels resistance. She should use more force to guide the catheter through his urethra. A. True B. False

1. apply sterile gloves 2. apply fenestrated drape 3. lubricate the catheter 4. hold penis with non dominant hand 5. clean penis with dominant hand 6. coil catheter in dominant hand 7.ask patient to bear down 8.advance catheter into penis Answer: 7 to 9 Rationale: In an adult male the catheter should be advanced 7 to 9 inches (17-22.5 cm) or until urine flows out of it. Answer: B Rationale: When resistance is felt while inserting a catheter, withdraw the catheter and do not force it. If there is resistance to the catheter insertion, have the patient take slow, deep breaths while slowly inserting the catheter. When urine appears, advance the catheter another 2.5 to 5 cm (1 to 2 inches).

A 70-year-old woman complains of involuntary passage of urine. The leakage of urine occurs in small amounts and is more frequent when she coughs. The nursing assessment reveals that the client is obese, has had three pregnancies, and has already gone through menopause. The nurse understands that she is at increased risk of developing urinary tract infection. What nursing interventions are helpful to prevent a urinary tract infection in the client?

1. emphasize cotton wear 2. promote complete emptying of bladder by double voiding 3. emphasize the importance of perineal hygiene

preventing infection in patients with catheters

1. follow hand hygiene 2. don't touch the spigot or get it contaminated 3. only use sterile technique to collect specimens from a closed drainage bag 4. drain all before exercise 5.empty q8h 6.tape and secure appropriately

A client who presents with dribbling of urine is diagnosed with stress incontinence. What else should the nurse include in the assessment of this client? Select all that apply.

1. height and weight-obesity 2. menopausal status-decrease in estrogen production, pelvic muscles weaken, leading to stress incontinence 3. pregnancy puts pressure on the bladder osteoarthritis does not

elimination changes that result from inability of the bladder to empty properly may cause?

1. incontinence 2. frequency 3. urgency 4. urinary retention 5. UTI

A client is experiencing difficulty in voiding. Which nursing interventions may help to stimulate the micturition reflex in the client? Select all that apply.

1. induce sound of running water. 2. stroke the inner aspect of the sensory nerves and promotes micturiton reflex 3. pour warm water over perineum 4. help the client assume normal voiding position

indications for catheter usage

1. intermittent catheterization →relieving discomfort or bladder distention, providing decompression →obtaining sterile urine specimen when clean catch specimen is unobtainable →assessing residual urine after urination -managing patients with spinal cord injuries, neuromuscular degeneration or incompetent bladders long term 2. short term indwelling catheterization -obstruction of urine outflow-prostate enlargement -surgical repair of bladder, urethra, surrounding structures -measurement of urinary output in critically ill -prevention of urethral obstruction from blood clots after genitourinary surgery -continuous or intermittent bladder irrigation 3. long term indwelling catheterization -severe urinary retention with recurrent episodes of UTI -skin rashes, ulcers, or wounds irritated by contact with urine -terminal illness when bed linen changes are painful for patients

helping patient with stress urinary incontinence to gain control over urination

1. learning exercises to strengthen the pelvic floor 2. intitating a toilet schedule on awakening, at least q22h during the day n evening, before bed, and q4h at night 3. avoiding overfilled bladder 4. minimize tea, coffee, caffeine drinks and alcohol 5. taking prescribed diuretic meds in the morning 6. following a weight-control program if obestiy is a problem that is causing increased abdominal pressure kegal and bladder retraining

What is the primary purpose of a soft, high-fiber diet immediately following a myocardial infarction (MI)?

1. maintain bowel health to decrease flatulence Dietary fiber is used to stimulate the wall of the intestinal tract to move the stool through and out of the rectum, without a client needing to bear down to force the stool out of the rectum. The Valsalva maneuver (straining with a closed glottis) causes bradycardia and decreases cardiac output, and should be avoided after an MI. Fiber does not get digested, it only aids in adding bulk to the stool. Fiber helps in lowering cholesterol but this action is not the primary purpose of it following an MI. Fiber does not decrease gas production.

A nurse is caring for a client who has undergone a surgery on the pharynx. The client has a small bore nasogastric tube. What are the purposes of a small bore nasogastric tube?

1. medication administration 2. enteral feeding

catheter care

1. need health care order 2. maintains a closed urinary drainage system 3. hang plastic bag on the bed frame or wheelchair 4. never on the bed rail 5. look for kinks and blockage in tubing 6. perineal hygiene ->3 times a day to prevent buildup of secretions or encrustation ->soap and water is effective 7. patients with catheters receive special care 3x a day and after defecation or bowel incontinence 8. daily intake of 2000-2500 mL

The patient is to have an intravenous pyelogram (IVP). Which of the following apply to this procedure? (Select all that apply.) Note any allergies. Monitor intake and output. Provide for perineal hygiene. Assess vital signs. Encourage fluids after the procedure.

1. note allergies 2. encourage fluids after the procedure The dye used in the procedure is iodine based. Assessing for history of any allergies can predict allergy to the dye used. Fluid intake dilutes and flushes the dye from the patient.

Alkaline phosphatase increases with

1. obstructive heptaolilary disease 2. hepatobiliary carcinomas 3. bone tumors 4. healing fractures

Correct order for irrigating a nasogastric tube (NG):

1. perform hand hygiene and apply clean gloves 2. clamp and disconnect the NG tube 3. insert tip of syringe into NG tube and slowly inject 30 mL of saline 4. slowly aspirate the syringe 5. reconnect the NG tube to suction

A nurse is caring for a client who has undergone a surgery on the pharynx. The client has a small bore nasogastric tube for enteral feeding. After an assessment, the health care provider orders removal of the tube. The nurse follows all the steps of the procedure to remove the tube. Arrange the steps of the procedure in the correct sequence.

1. perform hand hygiene and clean gloves 2. turn off suction and disconnect NG tube from the drainage bag or suctin 3. Remove tape of fixation device from bridge of nose and unpin tube from gown 4. stand on the clients right side if right-handed, left side if left-handed 5. hand the client facial tissue; place clean towel across chest 6. intruct client to take and hold a deep breath 7. clamp or kink tubing securely and pull tube out steadily and smoothly into towel in other hand 8.clean nares and provide mouth care 9. dispose of tube and drainage equipment in the proper container

A client is advised to have a barium swallow test. What instructions should the nurse provide to the client to prepare the client for the test? Select all that apply 1 Remove all jewelry before the test. 2 Don't eat anything after midnight. 3 Lie very still during the procedure. 4 Have a clear liquid diet 24 hours before the test. 5 Prepare the bowel using GoLytely

1. remove all jewelryt before the test 2. dont eat anything after midnight Barium swallow tests examine the structure and motility of the GI tract by using a contrast media like barium. The client needs to remove all jewelry or metallic objects from the body as they may interfere with the x-ray taken. The client should fast before the test, because the stomach and intestines should be empty for visualizing. It is not necessary to lie still during the procedure or to have a clear liquid diet for 24 hours before the test. Bowel preparation with GoLytely is performed for investigations like enteroclysis, colonoscopy, flexible sigmoidoscopy, and barium enema.

A client who is a smoker complains of involuntary passage of urine after a strong sense of urgency to void. What nursing interventions are helpful to this client? Select all that apply.

1. smoking cessation 2. antimuscarinic agents 3. behavioral interventions Urge incontinence is characterized by an involuntary passage of urine following a strong urge to void. Smoking can irritate the bladder and worsen the incontinence; therefore, the client should be instructed to quit smoking. Antimuscarinic agents help to prevent the involuntary contraction of the bladder muscles and prevent passage of urine. Behavioral interventions are helpful in making lifestyle changes to adjust for incontinence. Crede's method is helpful for clients with overflow incontinence. Intermittent catheterization is used when a client has urinary retention with overflow incontinence.

what 3 tasks does the stomach perform

1. storing swallowed food & liquid 2. mixing food, liquid & digestive juices 3. emptying its contents into the small intestine

A nurse is caring for a client admitted with diarrhea. What could be the possible causes of diarrhea in the client? Select all that apply. 1.Use of opioid drugs 2.Use of antibiotics 3.Food allergies 4.Prolonged stress 5.Hypothyroidism

1. use of antibiotics 2. food allergies 3. prolonged stress Use of antibiotics may cause diarrhea by disrupting the normal flora of the intestine. Food allergies and prolonged stress cause increased peristalsis resulting in diarrhea. Use of opioid drugs and hypothyroidism cause constipation by decreasing peristalsis.

A practitioner uses a urine specimen for culture and sensitivity via a straight catheter for a patient. What should the nurse do when collecting this urine specimen? 1.Use a sterile specimen container, 2.collect urine from the catheter port, 3.inflate the balloon with 10 mL of sterile water, 4.have the patient void before collecting the specimen

1.* A culture attempts to identify the micro organisms present in the urine, and a sensitivity study identifies the antibiotics that are effective against the isolated micro organisms. A sterile specimen container is used to prevent contamination of the specimen by micro organisms outside the body(exogenous) 2. The urine from straight catheter flows directly into the specimen container. Collecting a urine specimen from a catheter port is necessary when the patient has a urinary retention catheter. 3. A straight catheter has a single lumen for draining urine from the bladder. A straight catheter does not remain in the bladder and therefore does not have a 2nd lumen for water to be inserted into a balloon 4. This may result in no urine left in the bladder for the straight catheter to collect. A minimum of 3 mL of urine is necessary for a specimen for urine culture and sensitivity.

A nurse is taking care of a client who is bedridden. The nurse implements dietary modifications to relieve constipation. The client requests a bed pan. The nurse provides the client with a bed pan and ensures that the client is in a comfortable position to defecate. Arrange the steps for placing the bedridden client on a bedpan, in the correct sequence.

1.Ensure the client is positioned high in the bed. 2.Raise the client's head about 30 degrees. 3. provide support of upper torso 4.Ask the client to raise the hips by bending the knees and lifting the hips upward. Incorrect 5.Place a hand palm up under the client's sacrum, 6.resting the elbow on the mattress Slip the pan under the client.

Which statement by a patient with an ileostomy alert the nurse to the need for further education? 1."I don't expect to have much of a problem with fecal odor" 2. "I will have to take special precaution to protect my skin around the stoma" 3."I'm going to have to irragate my stoma so I have a bowel movement every morning" 4."I should avoid gas forming foods like beans to limit funny noises from the stoma"

1.The odor from drainage is minimal because fewer bacteria are present in the ileum compared with the large intestine. And ileostomy is an opening into the ileum (distal small intestine from the jejunum to the cecum) 2. cleansing the skin, skin barriers, and a well fitted appliance are precautions to protect the skin around the ileostomy stoma. The drainage from ileostomy contains enzymes that can damage the skin. 3.* this statement is inaccurate in relation to an ileostomy and indicates that the patient needs more teaching. And ileostomy produces liquid fecal drainage that is constant and cannot be regulated 4. An iliostomy stoma does not have a sphincter that can control the flow of flatus or drainage, resulting in noise

An elderly male client has been admitted to the hospital for a urinary tract infection. Which of the following physiological changes in the urinary system should the nurse teach the client about? Select all that apply.

1.Urinary retention increases risk of urinary infection. 2.Prostate enlargement may lead to urinary retention. 3.Ineffective bladder contraction leads to urinary retention. Urinary retention increases risk for bacterial growth and development of urinary infection. Prostate enlargement may lead to urinary retention by obstructing the flow of urine. Because the bladder cannot contract effectively, an older adult often retains urine after voiding. The older adult often experiences nocturia. Prostate enlargement increases urinary frequency due to incomplete voiding.

total bilirubin: 0.3-1 mg/dL

1.increased in hepatobiliary diseases (affecting the liver and bile, bile ducts, and gallbladder 2. increased in obstructions of bile ducts, 3. increased in certain anemia 4. increased in certain transfusion reactions

adults need to drink __to___ ml of water to avoid constipation.

1100-1400

how much should an adult need to drink a day

1100-1400 mL

After a transurethral prostatectomy a patient returns to his room with a triple-lumen indwelling catheter and continuous bladder irrigation. The irrigation is normal saline at 150 mL/hr. The nurse empties the drainage bag for a total of 2520 mL after an 8-hour period. How much of the total is urine output?

1320 The output is determined by calculating the amount of irrigation solution and subtracting that from the total output: 150 × 8 = 1200. Total output is 2520. 2520 - 1200 = 1320 urine output., 1320 mL, 1320 mL, The output is determined by calculating the amount of irrigation solution and subtracting that from the total output: 150 × 8 = 1200. Total output is 2520. 2520 - 1200 = 1320 urine output."

what is the normal amount of adult feces per day

150 g/day

After reviewing a patient's laboratory reports, the nurse concludes the patient has cholecystitis. Which laboratory finding enabled the nurse to reach this conclusion?

2. Increased amylase levels Cholecystitis causes inflammation of the gallbladder. Increased amylase levels are observed in patients with cholecystitis due to obstruction of the bile duct. Increased bilirubin levels indicate hepatobiliary disease. Increased alkaline phosphatase levels indicate hepatobiliary carcinomas and bone tumor. Increased carcinoembryonic antigen levels indicate cancer or inflammation of the gastrointestinal tract.

A nurse is caring for a client admitted with diarrhea. What could be the possible causes of diarrhea in the client? Select all that apply. 1. Use of opioid drugs Correct 2 Use of antibiotics Correct 3 Food allergies Correct 4 Prolonged stress 5 Hypothyroidism

2. Use of antibiotics 3. Food allergies 4. Prolonged stress

balloon size

3 ml (peds) 75 ml (large postoperative volume 5-30 ml most common 5 allows for optimal drainage

how many times a day does mass peristalsis occur

3 or 4 times per day

A 70-year-old woman complains of involuntary passage of urine. The leakage of urine occurs in small amounts and is more frequent when she coughs. What should the nurse teach the client about her disorder? Select all that apply 1. due to local irritation 2. nervous system disorders 3. weakness of muscles around urethra 4. stress incontinence 5. intra-abdominal pressure exceeds urethral resistance

3. weakness of muscles around urethra 4. stress incontinence 5. intra-abdominal pressure exceeds urethral resistance Involuntary voiding of urine on coughing occurs due to weakness of muscles around the urethra. It is also called stress incontinence. Stress incontinence occurs in older women when intra-abdominal pressure exceeds urethral resistance. Involuntary voiding occurs only when abdominal pressure rises above the urethral pressure. Local irritating factors and nervous system disorders usually lead to urge incontinence.

Normal value for alkaline phosphatase?

30-120 units/L

hyperactive bowel sounds occur how often?

35 or more per min. They occur with sm. intestine obstruction and inflammatory disorders.

How many times does an infant normally defecate in a day?

4-6 times if breast fed 1-3 times if bottle fed

normal Bowel sounds occur every __ to __ seconds.

5-15

Normal value for amylase?

60-120 somogyi units/dL

Opoid analgesics

<peristalsis= often results in constipation

antibiotics

>diarrhea

Nonsterroidal anti-inflammatory

>gi bleeding

aspirin

>gi bleeding

A nurse is caring for a client who has diarrhea for the past four days. When assessing a client, the nurse should expect which of the following findings? Select all that apply. A.Bradycardia. B.Hypotension. C.Fever. D.Poor skin turgor. E. Peripheral edema

A. Prolonged diarrhea is more likely to cause take a tachycardia than bradycardia. B.*Prolonged diarrhea lead to dehydration, which causes a decrease in blood pressure. C.*Prolonged diarrhea leads to dehydration, which causes fever. D. *Prolonged diarrhea leads to dehydration, which causes poor skin turgor. E. Peripheral edema results from a fluid overload. Prolonged diarrhea is more likely to cause a fluid deficit

A client who has an indwelling catheter reports I need to urinate. Which of the following interventions should the nurse perform? A. Check to see whether the catheter is patent B. Reassure the client that it is not possible for her to urinate C. Re-catheterize the bladder with a larger gauge catheter D. Collect a urine specimen for analysis

A. * A clogged or kinked catheter causes the bladder to fill and stimulates the need to urinate B. Reassuring the client that is not possible to urinate is a non-therapeutic response because it diminishes the clients concern C. There are less invasive approaches the nurse can take before replacing the catheter D. Although it may become necessary to collect a urine specimen, there is a simpler approach the nurse can take to assess and possibly resolve the clients problem

A nurses caring for a client who has a tracheostomy. Which of the following actions should the nurse take each time he provides a tracheostomy care? Select all that apply. A.Apply the oxygen source loosely if the SPO2 increases during the procedure. B.Use surgical asepsis to remove and clean the inner cannula. C.Clean the outer surfaces in a circular motion from the stoma site outward. D.Replace the tracheostomy ties with new ties. E. Cut a slit in gauze squares to place beneath the tube holder.

A. * The nurse must be prepared to provide supplemental oxygen in response to any decline in oxygenation saturation while performing tracheostomy care. B.*The nurse should use a sterile disposable tracheostomy cleaning kit or sterile supplies and maintain surgical asepsis throughout this part of the procedure. C. * this helps move mucus and contaminated material away from the stoma for easy removal. D.To help keep the skin clean and dry, the nurse should replace the tracheostomy ties if they are wet or soiled. There is a risk of two dislodgment with replacing the ties, so he should not replace them routinely. E.The nurse should use a commercially prepared tracheostomy dressing with a slit in it. Cutting gauze squares can loosen lint or cause fibers the client could aspirate

A provider prescribes a 24 hour urine collection for a client. Which of the following actions should the nurse take? A. Discard the first voiding B. Keep all voidings in a container at room temperature C. Ask the client to urinate and pour the urine into a specimen container D. Ask the client to urinate into the toilet ,stop midstream ,and finish urinating into the specimen container

A. * The nurse should discard the first voiding of the 24 hour urine specimen, and note the time B. The nurse should collect all voidings after that and keep them in a refrigerated container. C. For a urinalysis, the nurse should ask the client to urinate and pour the urine into a specimen container D. For a culture, the nurse should ask the client to urinate first into the toilet, then stop midstream, and finish urinating in the specimen container

A nurse educator on a medical unit is reviewing factors that increase the risk of urinary tract infections with a group of assistive personnel. Which of the following should be included in the review? Select all that apply. A. Having sexual intercourse on a frequent basis B. Lowering of testosterone levels C. Wiping from front to back D. The location of the vagina in relation to the anus E. Undergoing frequent catheterization

A. * having sexual intercourse on a frequent basis is a factor that increases the risk of UTI in both males and females B. The decrease in estrogen levels during menopause increases a woman's susceptibility to UTIs C. Wiping from front to back decreases a woman's risk of UTIs D. * The close proximity of the female urethra to the anus is a factor that increases the risk of UTIs E. * undergoing frequent catheterization and the use of indwelling catheters are risk factors for UTIs

A nurse is assessing a client who has an acute respiratory infection that puts her at risk for hypoxemia. Which of the following findings are early indications that should alert the nurse that the client is developing hypoxemia? Select all that apply. A.Restlessness. B.Tachypnea. C.Bradycardia. D.Confusion. E.Pallor

A. * restlessness is an early manifestation of hypoxemia, along with tachycardia, elevated blood pressure, use of accessory muscles, nasal flaring, tracheal tugging, and adventitious lung sounds B.* tachypnea is an early manifestation of hypoxemia, along with tachycardia, elevated blood pressure, use of accessory muscles, nasal flaring, tracheal tugging, and adventitious lung sounds. C. Bradycardia is a late manifestation of hypoxemia, along with stupor, cyanotic skin and mucous membranes, bradypnea, hypotension, and cardiac dysrhythmias. D. Confusion is a late manifestation of hypoxemia, along with stupor, cyanotic skin and mucous membranes, bradypnea, hypotension, and cardiac dysrhythmias. E.*Pallor is an early manifestation of hypoxemia, along with tachycardia, elevated blood pressure, use of accessory muscles, nasal flaring, tracheal talking, and adventitious lung sounds

A nurse is preparing to administer a cleansing enema to an adult client in preparation for a diagnostic procedure. Which of the following are appropriate steps for the nurse to take? Select all that apply. A. Warm the enema solution prior to installation. B.Position the client on the left side with the right leg flexed forward. C.Lubricate the rectal tube or nozzle. D.Slowly insert the rectal tube about 2 inches. E.Hang the enema container 24 inches above the clients anus

A. *The nurse should warm the enema solution because cold fluid can cause abdominal cramping and hot fluid can injure the intestinal mucosa. B.*This position allows a downward flow of solution by gravity along the natural anatomical curve of the sigmoid colon C. * lubrication prevents trauma or irritation to the rectal mucosa. D.This is an appropriate length of insertion for a child. For an adult client, the nurse should insert a tube 3 to 4 inches. E. The height of the fluid container affects the speed of installation. The maximum recommended height is 18 inches. Hanging the container higher than that could cause rapid installation and possibly painful distention of the colon

A nurse in a providers office is assessing a client who reports losing control of urine when ever she coughs, laughs, or sneezes. The client relates a history of three vaginal births, but no serious accidents or illnesses. Which of the following interventions are appropriate for helping to control or eliminate the clients incontinence? Select all that apply A. Limit total daily fluid intake B. Decrease or avoid caffeine C. Increase the intake of calcium supplements D.Avoid the intake of alcohol E. Use Crede maneuver

A. Because stress incontinence results from weak pelvic muscles and other structures, limiting fluid will not resolve the problem B. *Caffeine is a bladder irritant and can worsen stress incontinence C. Calcium has no effect on stress incontinence D. *Alcohol is a bladder irritant and can worsen stress incontinence E. The Crede maneuver helps manage reflex incontinence, not stress incontinence.

A nurse is preparing to initiate a bladder training program for a client who has a voiding disorder. Which of the following actions should the nurse take? Select all that apply A. Establish a schedule of voiding prior to meal times B. Have the client record voiding times C. Gradually increase the voiding intervals D. Reminded client to hold urine until next scheduled voiding time E. Provide a sterile container for voiding

A. Bladder training involves voiding at scheduled in frequent intervals and gradually increasing these intervals to four hours. Mealtimes are not regular, and the intervals maybe longer than every four hours B. * ask the client to keep track of voiding times is an appropriate nursing action C. * gradually increasing the voiding interval is an appropriate nursing action D. * The client should be reminded to hold urine until the next scheduled voiding time E. A sterile container is not used in a bladder training program

A nurse is caring for a client for whom a tap water enema is prescribed, to be repeated until the return is clear. Which of the following actions should the nurse take? A. Clarify the order with the provider. B. Explain the procedure to the client. C. Ensure that the tap water is not too hot. D. Keep the amount per enema to less than 1,000 mL.

A. Clarify the order with the provider. Tap water is a hypotonic solution that can cause water toxicity. It should not be repeated. The nurse should clarify the order with the provider. Explaining the procedure to the client, ensuring that the tap water is not too hot, and keeping the amount to less than 1,000 mL are not pertinent if the enema should not be repeated.

A nurse is talking with a client who reports constipation. When the nurse discusses dietary changes that can help prevent constipation, which of the following foods should the nurse recommend? A.Macaroni and cheese. B. Fresh food and whole wheat toast. C. Rice pudding and ripe bananas. D. Roast chicken and white rice

A. Macaroni and cheese is a low residue option that could actually worse and constipation B. * A high fiber diet promotes normal bowel elimination. The choice of fruit and toast is the highest fiber option C. Rice pudding and ripe bananas are low residue options that could actually worsen constipation. D. Roast chicken and white rice or low residue options that could actually worsen constipation

A nurse is caring for a client who will preform fecal occult blood testing at home. Which of the following information should the nurse include when explaining the procedure to the client? A.Eating more protein is optimal prior to testing. B.One stool specimen is sufficient for testing. C.A red color changes indicates a positive test. D.The specimen cannot be contaminated with urine.

A. Some proteins such as red meat, fish, and poultry can alter the test results. B.Three specimens from three different bowel movements are required. C.A blue color indicates blood in the stool. D. *For fecal occult blood testing at home, the stool specimens cannot be contaminated with water or urine

While a nurse is administering a cleansing enema, the client reports abdominal cramping. Which of the following is the appropriate intervention? A. Have a client hold his breath briefly. B.Discontinue the fluid installation. C.Remind the client that cramping is common at this time. D.Lower the enema fluid container.

A. Taking slow, deep breaths is more therapeutic for easing discomfort than holding the breath. B.The nurse should stop the installation if the clients abdomen becomes a rigid and distended or if the nurse notes bleeding from the rectum. C.This intervention is not therapeutic as it implies that the client must tolerate the discomfort and that the nurse cannot or will not do anything to ease it. D. * to relieve the clients discomfort, the nurse should slow the rate of installation by reducing the height of the enema solution container

A nurse is caring for a client who is having difficulty breathing. The client is lying in bed and is already receiving oxygen therapy via nasal cannula. Which of the following interventions is the nurses priority? A.Increase the oxygen flow. B.Assist the client to Fowlers position. C.Promote removal of pulmonary secretions. D.Attain a specimen for arterial blood gases

A. The client may need more oxygen, as hypoxemia may be the cause of his difficulty breathing. However, administering oxygen and adjusting the fraction of inspired oxygen requires the providers prescription after a careful assessment of the clients oxygenation status, there is a higher priority given the nature of the clients distress. B. * The priority action the nurse should take when using the airway, breathing, circulation approach to care delivery is to relieve the clients dyspnea. Fowlers position facilitates maximal long expansion and thus optimizing breathing. With the client in this position, the nurse can better assess and determine the cause of the clients dyspnea C. The client may need suction or expectoration, as pulmonary secretions may be the cause of his difficulty breathing. However, there is a higher priority given the nature of the clients distress. D.It is important to check the clients oxygenation status, and in many nursing situations, assessment precedes action, but there is a higher priority given the nature of the clients distress.

A nurse is preparing to perform endotracheal suctioning for a client. Which of the following are appropriate guidelines for the nurse to follow? Select all that apply. A.Apply suction while withdrawing the catheter. B.Perform suctioning on a routine basis, every 2 to 3 hours. C.Maintain medical asepsis during suctioning. D.Use a new catheter for each suctioning attempt. E.Limit suctioning to 2 to 3 attempts.

A.*The nurse should apply suction pressure only while withdrawing the catheter, not while inserting it. B. The nurse should not suction routinely, because suctioning is not without risk. It can cause mucosal damage, bleeding, and bronchospasm. C.Endotracheal suctioning requires surgical asepsis D. * The nurse should not reuse the suction catheter unless an in-line suctioning system is in place. E.*To prevent hypoxemia, the nurse should limit each section in session to 2 to 3 attempts and allow at least one minute between passes for ventilation and oxygenation.

A client has an alkaline phosphate level of 200 units/L. What can be interpreted from this laboratory value? The client has a normal level. The client has a transfusion reaction. The client has hepatobiliary carcinoma. The client has pancreatitis.

Alkaline phosphatase levels are indicative of hepatic and bone health. An increase in serum levels of the enzyme indicates hepatobiliary carcinoma. The normal level of alkaline phosphatase is 30-120 units/L. Bilirubin levels are elevated in transfusion reactions due to destruction of hemoglobin. Serum amylase levels indicate pancreatic function. Text Reference - p. 1098

The nurse recognizes that urinary elimination changes may occur even in healthy older adults because of which of the following? 1. The bladder distends and its capacity increases 2. Older adults ignore the need to void 3. Urine becomes more concentrated 4. The amount of urine retained after voiding increases

Answer : 4 Rationale: The capacity of the bladder may decrease with age but the muscle is weaker and can cause urine to be retained (Option 4). Older adults don't ignore the urge to void and may have difficulty getting to the toilet in time (Option 2)The kidney becomes less able to concentrate urine with age (Option3)

The nurse will need to assess the clients performance of clean intermittent self catheterization(CISC) for a client with which urinary diversion? 1. Ileal conduit 2. Kock pouch 3. Neobladder 4. Vesicostomy

Answer :2 Rationale: The ilio conduit and vesicostomy (opt1,4) are in continent urinary diversions, and clients are required to use an external ostomy appliance to contain the urine. Clients with a neobladder can control their voiding (opt3)

The nurse is most likely to report which finding to the primary care provider for a client who has an established colostomy? 1.The stoma extends 1/2 inch above the abdomen. 2.The skin under the appliance looks red briefly after removing the appliance. 3.The stoma color is a deep red purple. 4.An ascending colostomy just delivers liquid feces

Answer :3 Rationale : an established stoma should be dark pink like the color of the buccal mucosa and is slightly raised above the abdomen. The skin under the appliance may remain pink/red for a while after the adhesive is pulled off feces from an ascending ostomy are very liquid, less so from a transverse ostomy, and more solid from a descending or sigmoid stoma.

The catheter slips into the vagina during a straight catheterization of a female client. The nurse does which action? 1. Leaves the catheter in place and gets a new sterile catheter 2.Leaves the catheter in place and asks another nurse to attempt the procedure 3.Removes the catheter and redirects it to the urinary meatus 4.Removes the catheter, wipes it with a sterile gauze, and redirects it to the urinary meatus

Answer: 1 Rationale: The catheter in the vagina is contaminated and can't be reused.If left in place, it may help avoid mistaking the vaginal opening for the urinary meatus. A single failure to catheterize the meatus doesn't indicate that another nurse is needed although sometimes a second nurse can assist in visualization of the meatus (op2)

A client with a new stoma who has not had a bowel movement since surgery last week reports feeling nauseous. What is the appropriate nursing action? 1. Prepare to irrigate the colostomy. 2.After assessing the stoma and surrounding skin, notify the surgeon. 3.Assess bowel sounds and administer anti-emetic. 4.Administer a bulk forming laxative, and encourage increased fluids and exercise.

Answer: 2 Rationale: The client has assessment findings consistent with complications of surgery. Option one: irrigating the stoma is a dependent nursing action, and is also intervention without appropriate assessment. Option three: assessing the peristomal skin area is an independent action, but administering an anti-emetic is an intervention without appropriate assessment . antiemetics are generally ordered to treat immediate postoperative nausea, not several days postoperative. Option four: administering a bulk forming laxative to a nauseated postoperative client is contra indicated

Which action represents the appropriate nsg management of a client wearing a condom catheter? 1. Ensure that the tip of the penis fits snugly against the end of the condom 2. Check the penis for adequate circulation 30 min after applying 3. Chg the condom every 8 hours 4. Tape the collecting tube to the lower abd.

Answer: 2 Rationale: The penis & condom should be checked 1/2 hour after application to ensure that it's not too tight. A 1 in. space should be left btw the penis and the end of the condom (Opt1) The condom is chgd every 24h (opt3) and the tubing is taped to the leg or attached to a leg bag (opt4) An indwelling catheter is taped to the lower abd. or upper thigh

While a client with chest tubes is ambulating, the connection between the tube and the water seal dislodges. Which action by the nurse is most appropriate? 1. Assist the client to ambulate back to bed. 2.Reconnect to the tube to the water seal. 3.Assess the clients lung sounds with a stethoscope. 4.Have the client cough forcibly several times.

Answer: 2 Rationale: The tube should be reconnected to the water seal as quickly as possible. Assisting the client back to bed (option one) and assessing the clients lung(option three) are possible actions after the system is reconnected

An older client comes to the emergency department experiencing chest pain and shortness of breath. An arterial blood gases ordered. Which of the following ABG results indicate respiratory acidosis? 1. PH 7.5 for PaCO2 28 HCO3 22 2. PH 7.32 PaCO2 46 HCO3 24 3. PH 7.31 PaCO2 35 HCO3 20 4. PH 7.50 PaCO2 37 HCO3 28

Answer: 2 Rationale: because of the retention of CO2, the clinical profile of respiratory acidosis includes decreased pH PaCO2 of greater than 42 with varying levels of HCO3 related to hypoventilation.

An elderly man is admitted to the medical unit with a diagnosis of dehydration. Which sign or symptom is most representative of a sodium imbalance? 1. Hyperreflexia. 2.Mental confusion. 3.Irregular pulse. 4.Muscle weakness

Answer: 2 Rationale: sodium contributes to the function of neural tissue. Because calcium contributes to the function of voluntary muscle contraction, options one and four are more appropriate for calcium imbalances. Option three, because potassium and calcium contribute to cardiac function, irregular pulse is more likely to be associated with those alterations

A female client has a urinary tract infection. Which teaching points by the nurse should be helpful to the client? Select all that apply. 1. Limit fluids to avoid the burning sensation on urination 2. Review symptoms of UTI with the client 3. Wipe the perineal area from back to front 4. Wear cotton underclothes 5. Take baths rather than showers

Answer: 2,4 Rationale: option two validates the diagnosis. Cotton underwear promotes appropriate exposure to air, resulting in decreased bacterial growth(opt4). Increased fluids decrease concentration and irritation (opt1) The client should wipe the perineal area from front to back to prevent spread of bacteria from the rectal area to the urethra (opt3) showers reduce exposure of area to bacteria(opt5)

Which statement indicates a need for further teaching of a home care client with a long term indwelling catheter? 1."I will keep the collecting bag below the level of the bladder at all times" 2."Intake of cranberry juice may help decrease the risk of infection" 3."Soaking in a warm tub bath may ease the irritation associated with the catheter" 4."I should use clean tech. when emptying the collecting bag"

Answer: 3 Rationale: Soaking in a bathtub can increase the risk of exposure to bacteria. The bag should be below the level of the bladder to promote proper drainage.(Opt1)Intake of cranberry juice creates an environment nonconducive to infection(opt2)Clean tech. is appropriate for touching the exterior portions of the system(opt4)

To prevent postoperative complications, the nurse assist the client with coughing and deep breathing exercises. This is best accomplished by implementing which of the following? 1.Coughing exercises one hour before meals and deep breathing one hour after meals. 2.Forceful coughing as many times as tolerated. 3.Huff coughing every two hours or as needed. 4.Diaphragmatic and pursed lip breathing 5 to 10 times, four times a day

Answer: 3 Rationale: huff coughing helps keep the airways open and secretions mobilized. Huff coughing is an alternative for clients who are unable to perform a normal forceful cough (such as post operatively) deep breathing and coughing should be performed at the same time. Only at meal times is not sufficient (option one). Extended forceful coughing fatigues the client, especially post operatively (option two). Diaphragmatic and pursed lip breathing are techniques used for clients with obstructive airway disease (option four)

During shift report, the nurse learns that an older female client is unable to maintain continence after she senses the urge to void and becomes incontinent on the way to the bathroom. Which nursing diagnosis is most appropriate? 1.stress urinary incontinence 2. reflex urinary incontinence 3. functional urinary incontinence 4 urge urinary incontinence

Answer: 4 rationale: The key phrase is "the urge to void" option one occurs when the client coughs, sneezes, or jars the body, resulting in accidental loss of urine. Option two occurs with involuntary loss of urine at somewhat predictable intervals when a specific bladder volume is reached. Option three is involuntary loss of urine related to impaired function

A student nurse is assigned to care for a client with a sigmoidostomy. The student will assess which ostomy site? Referred to page 376 in fundamentals of nursing

Answer: 5 Rationale: option five is a sigmoid ostomy site. Option one is an ileostomy site, option two is an ascending colostomy, option three is transverse colostomy, and option four is descending colostomy

Clients should be taught that repeatedly ignoring the sensation of needing to defecate could result in which of the following? 1. Constipation 2.diarrhea 3.incontinence 4.hemorrhoids

Answer:1 Rationale: habitually ignoring the urge to defecate can lead to constipation through loss of the natural urge and the accumulation of feces. Diarrhea will not result-if anything, there is increased opportunity for water reabsorption because the stool remains in the colon, leading to firmer stool (opt2) ignoring the urge shows a strong voluntary sphincter, not a weak one that could result in incontinence (option three). Hemorrhoids would only occur only if severe drying out of the stool occurs, and thus repeated need to strain to pass stool (option four)

Which of the following is most likely to validate that a client is experiencing intestinal bleeding? 1. Large quantities of fat mixed with pale yellow liquid stool. 2.Brown, formed stool. 3.Semi soft tar colored stools. 4.Narrow, Pencil shaped stool

Answer:3 Rationale: blood in the upper G.I. tract is black and tarry. Option one can be a sign of malabsorbtion in an infant, option two is normal stool, and option four is characteristic of an obstructive condition of the rectum

The nurse administers an IV solution of D5 one half normal saline to a postoperative client. This is classified as what type of intravenous solution?

Answer: hypertonic

The nurse is preparing to perform tracheostomy care. Prior to beginning the procedure the nurse performs which action? 1.Tells the client to raise two fingers to indicate pain or distress. 2.Changes twill tape holding the tracheostomy and place. 3.Cleans the incision site. 4.Checks the tightness of the ties and knots

Answer:1 Rarionale: prior to starting the procedure, it is important to develop a means of communication by which the client can express pain or discomfort. The twill tape is not changed until after performing tracheostomy care (option two) cleaning the incision should be done after cleaning the inner cannula (option three) checking the tightness of the ties and knot is done after apply new twill tape (option four)

Which term does the nurse document to best describe a client experiencing shortness of breath while lying down who must assume an upright or sitting position to breed more comfortably and effectively? 1.Dyspnea 2.Hyperpnea 3.orthopnea 4.acapnea

Answer:3 Rationale: respiratory difficulty related to a reclining position without other physical alterations is defined as orthopnea

The nurse is planning to perform percussion and postural drainage. Which is an important aspect of planning the clients care? 1.Percussion and postural drainage should be done before lunch. 2.The order should be coughing, percussion, positioning, and then suctioning. 3.A good time to perform percussion and postural drainage is in the morning after breakfast when the client is well rested. 4.Percussion and postural drainage should always be preceded by three minutes of 100% oxygen.

Answer:1 Rationale: postural drainage result in expectoration of large amounts of mucus. Client sometimes ingest part of the secretions. The secretions may also produce an unpleasant taste in the oral cavity, which could result in nausea/vomiting. This procedure should be done on an empty stomach to decrease client discomfort

The nurse makes the assessment that which client has the greatest risk for a problem with the transport of oxygen from the lungs to the tissues? A client who has 1. anemia. 2.An infection. 3.A fractured rib. 4.A tumor of the medulla

Answer:1 Rationale: anemia is a condition of decreased red blood cells and decreased hemoglobin. Hemoglobin is how the oxygen molecules are transported to the tissues. Option two would depend on where the infection is located. Option three: a fractured rib would interrupt transport of oxygen from the atmosphere to the airways. Option four: damage to the medulla would interfere with neural stimulation of the respiratory system

Which nursing diagnoses is/are most applicable to a client with fecal incontinence? Select all that apply 1. Bowel incontinence. 2.Risk for deficient fluid volume. 3.Disturbed body image. 4Social isolation. 5Risk for impaired skin integrity

Answer:1,3,4,5 Rationale: option one is the most appropriate. The client is unable to decide when stool evacuation will occur. In option three, client thoughts about self may be altered if unable to control stool evacuation. In option five, increased tissue contact with fecal material may result in impairment. Option two is more appropriate for a client with diarrhea. Incontinence is the inability to control feces of normal consistency

Which statement provides evidence that an older adult who is prone to constipation is in need of further teaching? 1."I need to drink one and a half to 2 quarts of liquid each day" 2."I need to take a laxative such as milk of magnesium or if I don't have a BM every day" 3."if my bowel pattern changes on its own, I should call you" 4."eating my meals at regular times is likely to result in regular bowel movements"

Answer:2 Rationale: The standard of practice in assisting the older adults to maintain normal function of the gastrointestinal tract is regular ingestion of a well-balanced diet, adequate fluid intake, and regular exercise. If the bowel pattern is not regular with these activities, this abnormality should be reported. Stimulant laxatives can be very irritating and are not the preferred treatment for occasional constipation in older adults (opt2). In addition, a normal stool pattern for an older adult may not be daily elimination

An elderly nursing home resident has refused to eat or drink for several days and is admitted to the hospital. The nurse should expect which assessment finding? 1.Increase blood pressure. 2.Week, rapid pulse. 3.Moist mucous membranes. 4.Jugular vein distention.

Answer:2 Rationale: all other options are indicated of fluid volume excess. A client who has not eaten or drunk anything for several days would be experiencing fluid volume deficit.

A man brings his elderly wife to the emergency department. He states that she has been vomiting and has had diarrhea for the past two days. She appears lethargic and is complaining of leg cramps. What should the nurse do first? 1.Start an IV. 2.Review the results of serum electrolytes. 3.Offer the woman foods that are high in sodium and potassium content. 4.Administer an anti-a medic

Answer:2 Rationale: further assessment is needed to determine appropriate action. While the nurse may perform some of the interventions in options one, three, and four, assessment is needed initially.

Which goal is the most appropriate for clients with diarrhea related to ingestion of an antibiotic for an upper respiratory infection? 1. The client will wear a medical alert bracelet for antibiotic allergy. 2.The client will return to his or her previous fecal elimination pattern. 3.The client verbalizes the need to take an antidiarrheal medication PRN. 4.The client will increase intake of insoluble fiber such as grains, rice, and cereals.

Answer:2 Rationale: once the cause of diarrhea has been identified and corrected, the client to return to his or her previous elimination pattern. This is not an example of an allergy to the antibiotic but a common consequence of overgrowth of bowel organisms not killed by the drug (option one). Antidiarrheal medications are usually prescribed according to the number of stools, not routinely around the clock (option three). Increasing intake of soluble fiber such as oatmeal or potatoes may help absorb excess liquid and decrease the diarrhea, but insoluble fiber will not (option four)

Which client statement informs the nurse that his teaching about the proper use of an incentive spirometer was effective? 1."I should breathe out as fast and as hard as possible into the device" 2."I should inhale slowly and steadily to keep the balls up" 3."I should use the device three times a day, after meals" 4."the entire device should be washed thoroughly in sudsy water once a week"

Answer:2 Rationale: proper use of an SMI requires the client to take slow, steady inhalations, every hour or two, 5 to 10 reps each time. Only the mouthpiece can be successfully rinsed or wiped clean. The device should not be submerged in water (option four)

Which of the following behaviors indicates that the client on a bladder training program has met the expected outcomes? Select all that apply. 1. Voids each time there is an urge 2. Practices slow, deep breathing until the urge decreases 3. Uses adult diapers, for "just in case" 4. Drinks citrus juices and carbonated beverages 5. Performs pelvic muscle exercises

Answer:2 ,5 Rationale: it is important for the client to inhibit the urge to void sensation when a premature urge is experienced. Some clients may need diapers; this is not the best indicator of a successful program (opt3). Citrus juices may irritate the bladder (opt4). Carbonated beverages increase diuresis and the risk of incontinence (opt4)

Which focus is the nurse most likely to teach for a client with a flaccid bladder? 1. Habit training: attempt voiding at specific time periods 2 bladder training: delay voiding according to a pre-schedule timetable 3.Crede's maneuver: apply gentle manual pressure to the lower abdomen 4. Kegel exercises: contract the pelvic muscles

Answer:3 Rationale: because the bladder muscles will not contract to increase the intra-bladder pressure to promote urination, the process is initiated manually. Options one, two, and four: to promote continence bladder contractions are required for habit training, bladder training, and increasing the tone of the pelvic muscles.

Which action by the nurse represents proper nasopharyngeal/nasotracheal suctioning technique? 1. Lubricate the suction catheter with petroleum jelly before and between insertion. 2. Apply suction intermittently while inserting the suction catheter. 3.Rotate the catheter while applying suction. 4.Hyper oxygenate with 100% oxygen for 30 minutes before and after suctioning

Answer:3 Rationale: rotating the catheter prevents pulling of tissue into the opening on the catheter tip and the side. Suction catheters may only be lubricated with water or water-soluble lubricant and (petroleum jelly such as Vaseline has an oil base) (option one) no suction should ever be applied while the catheters being inserted because this can traumatize tissues (option two). The client should be hyper oxygenated for only a few minutes before and after suctioning and this is generally limited to clients who are intubated or have a tracheostomy (option four)

The nurse assesses a clients abdomen several days after abdominal surgery. It is firm, distended, and painful to palpate. The client reports feeling "bloated" the nurse consult with the surgeon, who orders an enema. The nurse prepares to give what kind of enema? 1. Soapsuds. 2Retention. 3.Return flow. 4.Oil retention.

Answer:3 Rationale: this provides relief of post operative flatus, stimulating bowel motility. Options one, two, and four manage constipation and do not provide flatus relief

A client with chronic pulmonary disease has a bluish tinge around the lips. The nurse charts which term to most accurately describe the clients condition? 1.Hypoxia 2.hypoxemia 3.dyspnea 4.cyanosis

Answer:4 Rationale: A bluish tinge to mucous membranes is called cyanosis. This is most accurate because it is what the nurse observes. The nurse can only observe signs/symptoms of hypoxia (option one). More information is needed to validate this conclusion. Hypoxemia requires blood oxygenation saturation data to be confirmed (option two) and dyspnea is difficulty breathing (option three)

Which client statement indicates a need for further teaching regarding treatment for hypokalemia? 1."I will use avocado in my salads" 2."I will be sure to check my heart rate before I take my digoxin" 3. "I will take my potassium in the morning after eating breakfast" 4."I will stop using my salt substitute "

Answer:4 Rationale: Salt substitutes contain potassium. The client can still use it with in reason. Option one avocado is higher in potassium the most foods. Option two, hypokalemia can potentiate digoxin toxicity and checking the pulse will help the client avoid this. Option three, it is important to take potassium with food to avoid gastric upset.

A client with emphysema is prescribed corticosteroid therapy on a short-term basis for acute bronchitis. The client asks the nurse how the steroids will help him. The nurse respond by saying that the corticosteroids will do which of the following? 1.Promote bronchodilation. 2.Help the client to cough. 3.Prevent respiratory infection. 4.Decrease inflammation in the airways

Answer:4 Rationale: glucocorticoids are prescribed because of their anti-inflammatory effect. Options one, two, and four are not achieved with glucocorticoids

A client is scheduled for a colonoscopy. The nurse will provide information to the client about which type of enema? 1.Oil retention 2. return flow 3high large volume 4low, small volume

Answer:4 Rationale: small volume enemas along with other preparations are used to prepare the client for this procedure. An oil retention enema is used to soften hard stool (option one) return flow enemas help expel flatus (option two) because of the risk of loss of fluid and electrolytes high,large volume enemas are seldom used (option three)

During assessment of the client with urinary incontinence, the nurse is most likely to assess for which of the following? Select all that apply. 1. Perineal skin irritation 2. Fluid intake of less than 1,500 mL/d 3. History of antihistamine intake 4. Hx of UTI 5. A fecal impaction

Answers: 1,2,4,5 Rationale: The perineum may become irritated by the frequent contact with urine (Opt.1). Normal fluid intake is at least 1,500 mL/d and clients often decrease their intake to try to minimize urine leakage (Opt.2) UTIs can contribute to incontinence. (Opt4) A fecal impaction can compress the urethra, which results in sm. amts of urine leakage (Opt5) Antihistamines can cause urinary retention rather than urinary incontinence. (Opt3)

The intake and output record of a client with a nasogastric tube that has been attached to suction for two days shows greater output and input. Which nursing diagnoses are most applicable? Select all that apply. 1. Deficient fluid volume. 2.Risk for deficient fluid volume. 3.Impaired oral mucous membranes. 4.Impaired gas exchange. 5.Decreased cardiac output

Answers: 1,3,5 Rationale: Options one, three, and five relate to fluid volume deficit. The data indicate an actual problem, which excludes option two. Option for relates more to fluid volume excess

Which medication has this action on the GI system (1091): Produce diarrhea by disrupting the normal bacterial flora in the GI tract.

Antibiotics

Which medication has this action on the GI system (1091): Inhibits gastric acid secretion and depress GI motility. Useful in treating hyperactive bowel disorders but may cause constipation.

Anticholinergic drugs such as atropine or glycopyrrolate (Robinul)

When assessing a 55-year-old client who is in the clinic for a routine physical, the nurse instructs the client about the need to obtain a stool specimen for guaiac fecal occult blood testing (gFOBT): 1 If client reports rectal bleeding. 2 When there is a family history of polyps. 3 As part of a routine examination for colon cancer. 4 If a palpable mass is detected on digital examination

As a part of the routine examination for colon cancer This is used as a dignostic screening tool for colon cancer by the American Cancer Society

Which medication has this action on the GI system (1091): Prostaglandin inhibitor; interferes with the formation and production of protective mucus and causes GI bleeding.

Aspirin

Which of the following medications listed in a client's medication history possibly causes gastrointestinal bleeding? Select all that apply. Aspirin Cathartics Antidiarrheal opiate agents Nonsteroidal antiinflammatory drugs (NSAIDs) New Choice

Aspirin Nonsteroidal antiinflammatory drugs (NSIADs) =side effects are rectal bleeding

A client presents with acute abdominal pain. The nurse suspects the presence of bowel necrosis in this client. What does the nurse expect to find in the laboratory test report? A. the bilrubin level is increased b. the serum amaylase is increased c. carcinoembryonic antigen level is increased d. the alkaline phosphatease levels are increased

B The presence of amylase indicates the function of the GI tract. Bowel necrosis is associated with increased serum amylase, due to pancreatic stimulation. Serum bilirubin levels indicate the functioning of the liver. Carcinoembryonic antigen indicates GI inflammation. Alkaline phosphatase becomes elevated when bone tumors are present.

What is the scale used to judge the form of stool

Bristol stool form scale

Which laxative/cathartic type is described: Not for pt. for whom large fluid intake is contraindicated. The drugs of choice for chronic constipation. Also relieves mild diarrhea. If treating diarrhea, add less water. Stretches intestinal wall to initiate peristalsis. The least irritating, most natural, and safest cathartics. Each dose should be followed with at least 8 oz of water

Bulk forming. Methylcellulose (Citrucel) Psllium (Metamucil, Naturcil) Polycarbophil (Fibercon) WARNING: if not mixed with enough liquid (at least 240 ml) or if swallowed too quickly can cause obstruction.

Carcinoembryonic antigen

CEA elevated in the presence of cancer or inflammation of the GI tract or hepatobiliary organs

A primary health care provider instructs the nurse to insert an indwelling urinary catheter in a client for 3 weeks. What type of catheter is the best choice for this client to prevent infection and promote comfort?

Catheters made of latex are suitable for clients who require catheterization for 3 weeks. These catheters are used for the short-term and may prevent infection if protected from contamination. Silicon and Teflon catheters are appropriate for clients who require catheterization for 2-3 months. Plastic catheters are appropriate for intermittent catheterization. Text Reference - p. 1062 latex

Since removal of the patient's Foley catheter, the patient has voided 50 to 100 mL every 2 to 3 hours. Which action should the nurse take first? 1 Check for bladder distention 2 Encourage fluid intake 3Obtain an order to recatheterize the patient 4Document the amount of each voiding for 24 hours

Check for bladder distention The patient may experience urinary retention after catheter removal. If amounts voided are small, checking for bladder distention is necessary.

A client complains that he is not able to pass urine completely. Even after voiding, the client does not feel that the bladder is empty. What tests can be done to assess the postvoid residual (PVR) in the client?

Correct 1 Portable noninvasive bladder ultrasound device Postvoid residual can be assessed using a portable noninvasive bladder ultrasound device. It helps to determine the amount of urine left in the bladder after voiding. A cystoscopy helps to visualize the structures of the urinary tract. An x-ray of the abdomen may show the condition of abdominal organs, but is not helpful in determining the residual urine left in the bladder. An intravenous pyelogram may help to determine the function of the kidneys, but does not help in determining postvoid residual.

The nurse is caring for a client with abdominal pain who is scheduled for a barium enema with air contrast. What information should the nurse give to the client before this procedure? Select all that apply. 1.The procedure will help in the examination of lower GI tract. 2.The client will have to be nil per os (nothing by mouth; NPO) after midnight. 3. No metallic objects are allowed during procedure 4.Light sedation is required for the procedure. 5.The client needs to lie very still during the procedure.

Correct 1 The procedure will help in the examination of lower GI tract. Correct 2 The client will have to be nil per os (nothing by mouth; NPO) after midnight. An x-ray film examination uses an opaque contrast medium and air that outlines the colon and rectum to examine the lower GI tract. Preparation includes nil-per-osl (NPO) after midnight, a bowel preparation such as magnesium citrate, and in some instances, enemas to empty out any remaining stool particles. Metallic objects do not interfere with the procedure; therefore, they do not need to be removed. Sedation is not required as it is not a painful procedure. There is no need for the client to lie very still as the procedure is not affected with client's movement.

The nurse recognizes which client needs to use a fracture pan for a bowel movement? a. client who is obese b. client experiencing confusion c. the clinet on bed rest d. a client recovering from hip surgery

D A fracture pan is used for a client with back or lower-extremity health issues. Because a fracture pan is shallow in comparison to a regular bedpan, the fracture pan prevents disturbing the client's body alignment. Text Reference - p. 1104

A nurse is caring for a client who will perform fecal occult blood testing at home. Which of the following should the nurse include when explaining the procedure to the client? A. Eating more protein is recommended prior to testing. B. One stool specimen is sufficient for testing. C. A red color change indicates a positive test. D. The specimen cannot be contaminated with urine.

D. The specimen cannot be contaminated with urine. For fecal occult blood testing at home, the stool specimens cannot be contaminated with water or urine; three specimens from three different bowel movements are required; some proteins such as red meat, fish, and poultry can alter the test results; and a blue color indicates a positive guaiac or presence of fecal occult blood not red. Chapter 45; Page 1046

Which laxative/cathartic type is described: Lowers surface tension of feces, allowing water and fat to penetrate. Increases secretion of water by intestine. For short-term use only to relieve straining on defecation. Of little value for chronic constipation.

Emollient or wetting. Docusate sodium (Colace, Correctol, Disonate) Docusate calcium (Surfak) Docusate potassium (Dialose)

do infants have slow or fast times of peristalsis

FAST-so cant it causes fast elimination. infants cant control defecation due to neuromuscular development

what does the large intestine absorb

H20, sodium, and chloride from digested food that has passed thru small intestine

what does the stomach produce

HCL, mucus, pepsin, & intrinsic factor

Why use tap-water enemas?

HYPOtonic and exerts an osmotic pressure lower than fluid in interstitial space. Stimulates defecation before large amounts of water leave the bowel. DO NOT repeat tap-water enemas because water toxicity or circulatory overload develops if they body absorbs large amounts of water (Page 1107) AND it is a hypotonic solution, thus it pulls electrolytes from the body into the fluid. This increases the risk of electrolyte imbalance (Lecture).

High enemas vs low enemas?

High enemas cleanse the entire colon. Low enemas cleanse ONLY rectum and sigmoid colon. Chapter 46; Page 1107

Which medication has this action on the GI system (1091): Suppress the secretion of HCl and interfere with the digestion of some foods

Histamine (H2) antagonists

The nurse receives a prescription to obtain a post-void residual for a client via catheterization. What is the best way to obtain this measurement?

Intermittent catheterization is used when evaluating the residual urine following urination. The investigation requires measurement of urine remaining in the bladder after voiding. Intermittent catheterization prevents the risk of infection. Long-term catheterization is done in clients with urinary retention. It may also be done for clients with recurrent episodes of urinary infections, skin breakdown, and terminal illness. Short-term catheterization is required for obstructive conditions, surgical repair of bladder and urethra, prevention of urethral obstruction, and bladder irrigation. There is no such thing as medium-term indwelling catheterization. Text Reference - p. 1061

Visually observable peristalsis indicates what?

Intestinal obstruction.

Which portion of the GI tract is affected here? These are normal age-related changes: (p 1090) Increase in pouches on weakened wall called Diverticulosis. Caused by weakened musculature. Constipation. Caused by decreased peristalsis. Missed defecation signal (increases risk for fecal incontinence). Caused by duller nerve sensations.

Lg. intestine.

Which laxative/cathartic type is described (1106): Coats fecal contents, allowing easier passage of stool. Reduces water absorption in colon. Prevents straining on defecation. Decreases absorption of fat-soluble vitamins (A, D, E, and K). Causes dangerous form of pneumonia if aspirated into lungs.

Lubricants. Mineral oil (Haley's MO, Petrogala Plain) WARNING: When taken with emollients, mineral oil increases risk for fat emboli.

MRI

NPO 4-6 hours before no metallic objects

Which enema is the safest to use?

NS enema

An alcoholic develops chronic pancreatitis. Which laboratory parameter is helpful in diagnosing pancreatitis?

Raised serum amylase

A client is admitted for lower gastrointestinal (GI) bleeding. What color of stool does the nurse anticipate the client to have?

Red-colored stool indicates lower GI bleeding.

do the elderly have a higher rate of fast peristalsis or slow

SLOW- systemic changes issues to slow it down. ex. arteriosclerosis <mesenteric blood flow <slows all blood flow <which slows absorption Peristalsis slows and esophageal emptying slows

Which laxative/cathartic type is described: Contains a salt preparation not absorbed by the intestines. Has an osmotic effect that increases pressure in the bowel to act as a stimulant for peristalsis. Only for acute emptying of the bowel (such as for diagnostic testing, poisoning, or acute constipation) Not for long-term use not for pt. with kidney dysfunctions (allows a toxic build up of magnesium to occur). Not for pt. on fluid restrictions.

Saline. Magnesium citrate or citrate of magnesia. magnesium hydroxide (milk of magnesia) Sodium phosphate (Fleet enema, Fleet phosphosoda)

A client reports having the urge to void but urine starts leaking before she reaches the bathroom. Which treatment strategies would be helpful for the client? Select all that apply. Scheduled toileting Absorbent products Electrical stimulation Clothing modification Antimuscarinic agents

Scheduled toileting Absorbent products Clothing modification Functional incontinence is characterized by the inability to reach the bathroom in time. Scheduled toileting involves teaching the client to void at specified times so that there is no urgency. Use of absorbent products helps prevent soiling of clothes. Clothing can be modified to make it easier to remove when there is an urgency to void. Electrical stimulation is helpful for clients with stress incontinence. Antimuscarinic agents are helpful for clients with urge incontinence. Text Reference - p. 1060

What does this indicate? Translucent mucus in stool

Spastic constipation, colitis, excessive straining

What are soapsuds enema?

Stimulating peristalsis through intestinal irritation. USE ONLY PURE CASTILE SOAP because other harsher soap products are contraindicated, as they can cause bowel inflammation. Caution in pregnant women and older adults because they cause electrolyte imbalance or damage the intestinal mucosa. Chapter 46; Page 1107

Which portion of the GI tract is affected here? These are normal age-related changes: (p 1090) Decrease in acid secretions, which can cause malabsorption of iron. Caused by degeneration of mucosa. Decreased motor activity. Caused by delayed emptying, which leads to fewer hunger contractions. Decreased mucosa thickness. Caused by loss of parietal cells.

Stomach. Parietal cells secrete intrinsic factor, so a decrease would lead to less B12 absorption.

A client is suffering from chronic constipation but has no other symptoms. Which medication would provide the most relief for the client's constipation? a. castor oil b. mineral oil c. polycarbophil d. docusate sodium

The drug of choice for managing chronic constipation in a client who is hemodynamically stable is a bulk-forming substance such as polycarbophil. Castor oil causes cramping, as well as an imbalance in fluid and electrolytes and should be avoided for long-term use. Mineral oil causes nutritional deficiency and on aspiration causes pneumonia, and should be avoided. Docusate sodium is suitable for short-term therapy.

A nurse is reviewing the records of a client who had chronic constipation. The record states that the client has type 3 stools according to the Bristol Stool Form Scale. What would be the characteristic of the stool? 1. like a saugage but with cracks on the surface 2. like a sausage or snake, smooth and soft 3. soft blobs with clear-cut edges 4. sausage shaped but lumpy

Type 3 stools are like a sausage but with cracks on the surface. Type 4 stools are like a sausage or a snake, and are smooth and soft. Types 5 stools are soft blobs with clear-cut edges. Type 2 stools are sausage-shaped but lumpy. Text Reference - p. 1092

list GI diseases associated with stress

Ulcerative colitis, IBS, gastric and duodenal ulcers, Crohns

The nurse is taking a health history of a newly admitted client with a diagnosis Rule/out bowel obstruction. Which of the following is the priority question to ask the client?

When was the last time you moved you bowels - lack of bowel movement is a sign of bowel obstruction and is a medical emergency

celiac disease

a hypersensitivity to protein in certain cereal grains and gluten

Diseases of the GI tract associated with stress

gastric and duodenal ulcers Crohns disease ulcerative colitis IBS

what does chime do

mixes with digestive juices (bile and amylase).

a blue color on the fecal occult blood test indicates?

a positive result

which post operative medication is most likely to contribute to constipation

morphine sulfate an opoid analgesic

Where does digestion begin

mouth

leakage around the catheter

a change in lumen size or use of antipasmodic meds is necessary

abnormal characteristics of stool: color

absense of bile=white or clay iron ingestion or upper gastrointestinal bleeding=black or tarry red=lower GI bleeding, hemorrhoids pale with fat=malabsorption of fat transulent mucus=spastic constipation, colitis, excessive straining bloody mucus: blood in feces, inflammation, infection

during emotional stress the digestive process is

accelared, and peristalsis is increased

Factors affecting bowel elimination

age fluid intake psychological factors position during defecation pregnancy meds, laxatives and cathartics diet physical activity pain surgery and anthesia diagnostic test

colonoscopy

an endoscope exam of the entire colon uses a colonoscope inserted into the rectum prep: similar to barium enema; clear liquids the day before and then some form of bowel cleanser such as GoLytely. Enemas till clear

amylase

an ezyme produced in the pancreas, and helps the digestion process

Slow peristalsis =?

constipation

why you only use sterile water to inflate the balloon

because saline crystallizes, resulting in incomplete deflation of the balloon at the time of the removal

what does the colon secrete

bicarbonate in exchange for chloride

white-gray-glay colored

biliary obstruction or a lack of bile production

What does this indicate? bloody mucus in stool

blood in feces, or infection

barium enema, colonoscopy and flexible sigmoidoscopy require what bowel prep

bowel preparation with magnesium citrate

black or tarry stools

consumption of iron

abnormal characteristics of stool: shape

narrow, pencil shaped: obstruction, rapid peristalsis

A client who visited the hospital for routine laboratory tests has been found to have an increased serum carcinoembryonic antigen. Which conditions are suspected in the client and require further assessment? Select all that apply. a. bowel necrosis b. diabetic ketoacidosis c. inflammation of GI tract d. inflammation of hepatobiliary organs e. transfusion reaction

c. inflammation of GI tract D. inflammation of hepatobilary organs *Carcinoembryonic antigen* increases in inflammatory disorders of the gastrointestinal tract and with inflammation of the hepatobiliary organs. Therefore, the client requires evaluation for these disorders. *Serum amylase* is a marker for GI function. *Bowel necrosis* and *diabetic ketoacidosis* are associated with increased serum amylase and not carcinoembryonic antigen. A transfusion reaction is associated with increased serum bilirubin due to destruction of hemoglobin, and not carcinoembryonic antigen.

Chronic use of __ causes the lg intestine to become less responsive to stimulation by __.

cathartics, laxatives

clostridium difficile (c diff)

causative agent of diarrhea. its caused by 1. factors that cause an overgrowth of c. diff 2. and by contact w/c. diff. antibiotics, chemo, and invasive bowel procedures(colonoscopy) may cause an overgrowth of c. diff. soap and water with bleach gets rid of it. the ELISA is the test for c diff

what are the 3 sections of the large intestine

cecum colon rectum

what happens if peristalsis is slow

chyme is hardened-constipation

what happens if peristalsis is fast

chyme is watery-diarrhea

foods that can create an environment in the bladder more prone to infection

citrus juices-orange, grapefruit, pineapple

An elderly African American reports a change in bowel habits with rectal bleeding and a sense of incomplete bowel evacuation. What disorder does the nurse suspect in this client? 1. infection 2. colon cancer 3. irritable bowel syndrome 4. inflammatory bowel syndrome

colon cancer Age and race are two factors that can indicate whether a client is at an increased risk of developing colon cancer. In this case, the client is an elderly African American. Statistics show that African Americans have a higher risk of developing colon cancer. In addition, the client's presentation of change in bowel habits, rectal bleeding, and sense of incomplete evacuation of bowel are warning signs of colon cancer, and the nurse should evaluate the client for this condition. Infections are usually present with diarrhea which may be associated with blood. Irritable bowel syndrome and inflammatory bowel disorders are associated with abdominal pain. Text Reference - p. 1100

normal characteristics of stool

color-brown odor-pungent consistency- soft, formed freq- infant 4-6 times adult-2-3 times a week amount-150g/day shape-resembles diameter of rectum constituents- dead bacteria, fat, pile pigement, cells lining intesting mucosa, water, undigested food

Which portion of the GI tract is affected here? These are normal age-related changes: (p 1090) Reduced motility, esp. in lower 3rd. Caused by degeneration of neural cells.

esophagus

he nurse is analyzing the fecal characteristics of a client. Which substances indicate an abnormality? Select all that apply. Excess fat Blood Mucus Bile pigment Dead bacteria

excess fat, blood, mucus The presence of excess fat is an abnormal finding and indicates liver dysfunction. The presence of blood and mucus in the stool are an abnormal finding and are associated with internal bleeding, infection, and inflammation. Bile pigment and dead bacteria are normal constituents of the fecal matter.

What foods help bowel movement

fiber, vegetables, fruit, whole grains gas-producing foods: onions, cauliflower, and beans

TYPE 6 stool

fluffy pieces with ragged edges a mushy stool

fluid intake and movement of bowels

fluid liquefies intestinal contents, easing its passage. reduced fluid intake slows passage and results in hardening of stool

What happens in the esophagus

food passes thru upper esophageal sphincter-a circular muscle that stops air from entering the esophagus & food from refluxing into the throat. As food moves down the esophagus it reaches the cardiac(or lower) esophageal sphincter. The sphincter prevents reflux of stomach contents back into esophagus

condom catheter

for incontinent or comatose men who still have complete and spontaneous bladder emptying. never use standard adhesive tape

promotion of omfort

frequent or unpredictable voiding, dysuria, and painful distention are sources of discomfort -comfort: clean, dry clothing -protective pad analgesics act on the urthral and bladder mucosa-relieve dysuria inflamed urthra, local discomfort: sitz bath

cause of hemorrhoids

hemorrhoids are dilated, engorged veins in the lining of the rectum. 1.increased venous pressure from straining at defecation 2. pregnancy 3.heart failure 4. chronic liver disease

Citrucel Metamucil/fiber con Colace/surfak/dialose saline milk of magnesa

high fiber less irritating drug of choice for chronic constipation stool softeners-short term therapy-little value for chronic constip. acute emptying of bowel-not for long term

what does chyme enter the large intestine through

ileocecal valve

A client who has malabsorption syndrome asks the nurse about the process of nutrient absorption. What response should the nurse give the client? Select all that apply.

ileum: absorbs fat and bile salts the jejunum absorbs carbs and proteins nutrients are absorbed into the blood vessels

where does resorption take place

in the small intestine. its so efficient in the small intestine that by the time chyme reaches the end of it is in pastelike form

does stress increase or decrease perstalsis

increases

Age related factors affecting bowel elimination

infant: -smaller stomach capacity -less secretion of digestive enzymes -rapid peristalsis -unable to control defacation because of lack of neuromuscular development older adults: loss muscle tone in anal and perineal sphinter never impulses to the anal region slow

What does this indicate? noxious change in the odor of feces

infection o r blood in feces

abnormal characteristics of stool: frequency

more than 3x a day; less than once a week

removal of indwelling catheter

insert syringe into the balloon injection port. slowly withdraw all the solution remains (partially inflated balloon traumatizes the urthral canal as the catheter is removed. After deflation-inform pt that they will feel *burning* sensation. pull smoothly and slowly noting the first voiding after removal and document the time and amount of voiding for the next 24 hr -if amounts small-frequent assessments made -if 4 hours have elasped without voiding or pt feels discomfort-necessary to reinsert the cath

abnormal characteristics of stool: blood, pus, foreign body, mucus, worms

internal bleeding, infection, swallowed objects, irritation, inflammation

what causes constipation

intestinal motility slows fecal mass is exposed to intestinal wall over time most water has been absorbed so it cant soften or lube

What is secreted by the stomach that is essential for vit. B12 absorption?

intrinsic factor (1088)

What does this indicate? Melena (black or tarry) stool

iron ingestion or upper GI bleeding

cystitis

irritated bladder causes a frequent and urgent sensation of the need to void. urine is cloudy bc of bacterial and white cells

mr stevens had a total intake of 1000mL iv fluids. what impact does that intake have on his bowel pattern

it is inadequate, contributes to constipation

lactose intolerance

lacks enzyme to breakdown the sugars in milk

what is the primary organ of bowel elimination

large intestine

Normal value for carcinoembryonic antigen (CEA)?

less than 5 ng/mL

Hypoactive bowel sounds occur how often?

less than 5 sounds per min

A client is being assessed for a possible urinary tract infection (UTI). Before sending a urinalysis specimen to the laboratory, a nurse collects a small amount of urine to perform a dipstick test. If the client has a UTI, which component should be detected in the urine? 1.protein 2. glucose 3. ketones 4. leukocytes

leukocytes A dipstick test is performed in the health care provider's office to test for different components. In this case, the health care provider tests white blood cells or leukocytes, which indicate an infection. Protein is detected in clients with nephropathy. Glucose is detected in clients with diabetes mellitus. Ketones are detected in clients with poorly controlled diabetes, starvation, and dehydration.

TYPE 3 stool

like a sausage but with cracks on the surface

TYPE 4 stool

like a sausage or snake-smooth and soft

abnormal characteristics of stool: consistency

liquid: diarrhea, reduced absorption hard: constipation

Which portion of the GI tract is affected here? These are normal age-related changes: (p 1090) Decreased size causes reduced storage capacity and ability to synthesize protein and metabolize medications.

liver

pure silicon

long term use-2 or 3 months.

What does this indicate? red stool

lower GI bleeding or hemorrhoids

dysuria

lower UTIs have pain or buring dduring urniation as urine flows over inflamed tissues -fever, chills, nausea, comiting, and malaise

What does this indicate? Pale stool with fat

malabsorption of fat

abnormal characteristics of stool: excess fat

malabsorption syndrome, enteritis, pancreatic disease, surgical resectinon of intestine

fecal occult blood test

measures microscopic amounts of blood in the feces

selecting size of catheter

men-16-18 Fr women- 14-16 Fr children-8-10 Fr

what does mucus in the stomach do

protects the stomach from acidity & enzymes

what does pepsin & HCL digest

protein

Carminative enemas?

provide relief from gaseous distention

self catheterization

pt with chronic disorders-spinal injuries teach pt the structures of urinary tract, clean vs sterile, importance of adequeate fluid intake, freqency of self-catherization 4-6 times a day with volumes of 400-500mL

habit training

pt with functional incontinence helps improve voluntary control -pateint establishes the pattern by documenting episodes of incontinence and scheduling voiding opportunities just before the urge time interval. goal: keep pt dry

Who is at most risk for impaction?

pts who are debilitated, confused, or unconscious.

What does this indicate? Narrow or pencil shaped stool

rapid peristalsis or obstruction

___ peristalsis occurs in infants

rapid. They have small stomachs and less secretion of digestive enzymes

A nurse is teaching a group of people regarding colon cancer. Which factors should the nurse explain as warning signs of colon cancer? Select all that apply. Rectal bleeding Obesity and inactivity Change in bowel habits Older than 50 years of age Having a family history of colon cancer

rectal bleeding change in bowel habits sensation of incomplete bowel evacuation

lower GI tract present stools

red stools

A client is advised to have a barium swallow test. What instructions should the nurse provide to the client to prepare the client for the test? Select all that apply. a.Remove all jewelry before the test. b.Don't eat anything after midnight. c,Lie very still during the procedure. d.Have a clear liquid diet 24 hours before the test. e.Prepare the bowel using GoLytely.

remove all jewelry before the test dont eat anything after midnight Barium swallow tests examine the structure and motility of the GI tract by using a contrast media like barium. The client needs to remove all jewelry or metallic objects from the body as they may interfere with the x-ray taken. The client should fast before the test, because the stomach and intestines should be empty for visualizing. It is not necessary to lie still during the procedure or to have a clear liquid diet for 24 hours before the test. Bowel preparation with GoLytely is performed for investigations like enteroclysis, colonoscopy, flexible sigmoidoscopy, and barium enema.

A client is advised to undergo dialysis. Which conditions are the indications for dialysis? Select all that apply.

renal failure that does not respond to conservative therapy worsening or uremic symptoms associated iwth renal failure severe electrolyte and fluid imbalance Dialysis is performed to decrease the fluid overload, and eliminate the metabolic toxins. It is indicated in renal failure that does not respond to medications and dietary changes. If the uremic symptoms associated with renal failure worsen, dialysis is performed to prevent complications. Severe electrolyte and fluid imbalance which does not respond to simple therapies would need dialysis for correction. Multiple sclerosis with damage to the nerve that controls bladder may result in incontinence. It can be managed by catheterization. Decreased blood urea nitrogen is not an indication of dialysis as it does not indicate renal impairment.

A client's urinalysis shows the presence of casts. Based on this laboratory data, what should the nurse suspect? 1. protein-calorie malnutrtion 2. renal injury 3. end stage renal disease 4. renal stone formation

renal injury The increased presence of granular casts is always an abnormal finding and is usually indicative of renal injury. Protein-calorie malnutrition is characterized by the presence of ketone bodies in the urine, not casts. Increased levels of urea and creatinine indicate end-stage renal disease. The presence of excess crystals predisposes a client to the development of renal stones, not casts.

magnesium citrate

saline purgative, fast acting and suitable for managing poisoning.

how do you treat flatulence

same as things used to promote defecation

Bristol Stool Scale TYPE 2

sausage-shaped but lumpy


संबंधित स्टडी सेट्स

Google Maps Test 50 Questions, Multiple Choice

View Set

Chapter 18: The cardiovascular System: the heart (book)

View Set

APES Exam Unit 5: Water and Land Use

View Set